Sie sind auf Seite 1von 143

1

NURSING MANAGEMENT AND LEADERSHIP

 Nursing Leadership and Management


 Leadership
 Delegation
 Chain of Command
 Organizational Structure
 Types of Leaders

1. Ms. Caputo is newly-promoted to a patient care manager position. She updates her knowledge on the
theories in management and leadership in order to become effective in her new role. She learns that some
managers have low concern for services and high concern for staff. Which style of management refers to
this?

A. Organization Man
B. Impoverished Management
C. Country Club Management
D. Team Management

2. Her former manager demonstrated passion for serving her staff rather than being served. She takes
time to listen, prefers to be a teacher first before being a leader, which is characteristic of

A. Transformational leader
B. Transactional leader
C. Servant leader
D. Charismatic leader

3. On the other hand, Ms. Caputo notices that the Chief Nurse Executive has charismatic leadership style.
Which of the following behaviors best describes this style?

A. Possesses inspirational quality that makes followers gets attracted of him and regards him with reverence
B. Acts as he does because he expects that his behavior will yield positive results
C. Uses visioning as the core of his leadership
D. Matches his leadership style to the situation at hand.
2

4. Which of the following conclusions of Ms. Caputo about leadership characteristics is TRUE?

A. There is a high correlation between the communication skills of a leader and the ability to get the job done.
B. A manager is effective when he has the ability to plan well.
C. Assessment of personal traits is a reliable tool for predicting a manager’s potential.
D. There is good evidence that certain personal qualities favor success in managerial role.

5. She reads about Path Goal theory. Which of the following behaviors is manifested by the leader who
uses this theory?

A. Recognizes staff for going beyond expectations by giving them citations


B. Challenges the staff to take individual accountability for their own practice
C. Admonishes staff for being laggards.
D. Reminds staff about the sanctions for non performance.

6. One leadership theory states that “leaders are born and not made,” which refers to which of the
following theories?

A. Trait
B. Charismatic
C. Great Man
D. Situational

7. She came across a theory which states that the leadership style is effective dependent on the situation.
Which of the following styles best fits a situation when the followers are self-directed, experts and are
matured individuals?

A. Democratic
B. Authoritarian
C. Laissez faire
D. Bureaucratic

8. She surfs the internet for more information about leadership styles. She reads about shared leadership
as a practice in some magnet hospitals. Which of the following describes this style of leadership?
3

A. Leadership behavior is generally determined by the relationship between the leader’s personality and the
specific situation
B. Leaders believe that people are basically good and need not be closely controlled
C. Leaders rely heavily on visioning and inspire members to achieve results
D. Leadership is shared at the point of care.

9. Ms. Caputo learns that some leaders are transactional leaders. Which of the following does NOT
characterize a transactional leader?

A. Focuses on management tasks


B. Is a caretaker
C. Uses trade-offs to meet goals
D. Inspires others with vision

10. She finds out that some managers have benevolent-authoritative style of management. Which of the
following behaviors will she exhibit most likely?

A. Have condescending trust and confidence in their subordinates


B. Gives economic or ego awards
C. Communicates downward to the staff
D. Allows decision making among subordinates

11. Henry is a Unit Manager I the Medical Unit. He is not satisfied with the way things are going in his
unit. Patient satisfaction rate is 60% for two consecutive months and staff morale is at its lowest. He
decides to plan and initiate changes that will push for a turnaround in the condition of the unit. Which of
the following actions is a priority for Henry?

A. Call for a staff meeting and take this up in the agenda.


B. Seek help from her manager.
C. Develop a strategic action on how to deal with these concerns.
D. Ignore the issues since these will be resolved naturally.

12. He knows that there are external forces that influence changes in his unit. Which of the following is
NOT an external force?
4

A. Memo from the CEO to cut down on electrical consumption


B. Demands of the labor sector to increase wages
C. Low morale of staff in his unit
D. Exacting regulatory and accreditation standards

13. After discussing the possible effects of the low patient satisfaction rate, the staff started to list down
possible strategies to solve the problems head-on. Should they decide to vote on the best change strategy,
which of the following strategies is referred to this?

A. Collaboration
B. Majority rule
C. Dominance
D. Compromise

14. One staff suggests that they review the pattern of nursing care that they are using, which is described
as a:

A. job description
B. system used to deliver care
C. manual of procedure
D. rules to be followed

15. Which of the following is TRUE about functional nursing?

A. Concentrates on tasks and activities


B. Emphasizes use of group collaboration
C. One-to-one nurse-patient ratio
D. Provides continuous, coordinated and comprehensive nursing services

16. Functional nursing has some advantages, which one is an EXCEPTION?

A. Psychological and sociological needs are emphasized.


B. Great control of work activities.
C. Most economical way of delivering nursing services.
D. Workers feel secure in dependent role
5

17. He raised the issue on giving priority to patient needs. Which of the following offers the best way for
setting priority?

A. Assessing nursing needs and problems


B. Giving instructions on how nursing care needs are to be met
C. Controlling and evaluating the delivery of nursing care
D. Assigning safe nurse: patient ratio

18. Which of the following is the best guarantee that the patient’s priority needs are met?

A. Checking with the relative of the patient


B. Preparing a nursing care plan in collaboration with the patient
C. Consulting with the physician
D. Coordinating with other members of the team

19. When Henry uses team nursing as a care delivery system, he and his team need to assess the priority
of care for a group of patients, which of the following should be a priority?

A. Each patient as listed on the worksheet


B. Patients who needs least care
C. Medications and treatments required for all patients
D. Patients who need the most care

20. He is hopeful that his unit will make a big turnaround in the succeeding months. Which of the
following actions of Henry demonstrates that he has reached the third stage of change?

A. Wonders why things are not what it used to be


B. Finds solutions to the problems
C. Integrate the solutions to his day-to-day activities
D. Selects the best change strategy

21. Joey is a newly-appointed nurse manager of The Holy Spirit Medical Center, a tertiary hospital
located within the heart of the metropolis. He thinks of scheduling planning workshop with his staff in
order to ensure an effective and efficient management of the department. Should he decide to conduct a
strategic planning workshop, which of the following is NOT a characteristic of this activity?
6

A. Long-term goal-setting
B. Extends to 3-5 years in the future
C. Focuses on routine tasks
D. Determines directions of the organization

22. Which of the following statements refer to the vision of the hospital?

A. The Holy Spirit Medical Center is a trendsetter in tertiary health care in the next five year
B. The officers and staff of The Holy Spirit Medical Center believe in the unique nature of the human person
C. All the nurses shall undergo continuing competency training program.
D. The Holy Spirit Medical Center aims to provide a patient-centered care in a total healing environment.

23. The statement, “The Holy Spirit Medical Center aims to provide patient-centered care in a total
healing environment” refers to which of the following?

A. Vision
B. Goal
C. Philosophy
D. Mission

24. Joey plans to revisit the organizational chart of the department. He plans to create a new position of a
Patient Educator who has a coordinating relationship with the head nurse in the unit. Which of the
following will likely depict this organizational relationship?

A. Box
B. Solid line
C. Broken line
D. Dotted line

25. He likewise stresses the need for all the employees to follow orders and instructions from him and not
from anyone else. Which of the following principles does he refer to?

A. Scalar chain
B. Discipline
7

C. Unity of command
D. Order

26. Joey orients his staff on the patterns of reporting relationship throughout the organization. Which of
the following principles refer to this?

A. Span of control
B. Hierarchy
C. Esprit d’ corps
D. Unity of direction

27. He emphasizes to the team that they need to put their efforts together towards the attainment of the
goals of the program. Which of the following principles refers to this?

A. Span of control
B. Unity of direction
C. Unity of command
D. Command responsibility

28. Joey stresses the importance of promoting ‘esprit d corps’ among the members of the unit. Which of
the following remarks of the staff indicates that they understand what he pointed out?

A. “Let’s work together in harmony; we need to be supportive of one another”


B. “In order that we achieve the same results; we must all follow the directives of Julius and not from other
managers.”
C. “We will ensure that all the resources we need are available when needed.”
D. “We need to put our efforts together in order to raise the bar of excellence in the care we provide to all our
patients.”

29. He discusses the goal of the department. Which of the following statements is a goal?

A. Increase the patient satisfaction rate


B. Eliminate the incidence of delayed administration of medications
C. Establish rapport with patients.
D. Reduce response time to two minutes.
8

30. He wants to influence the customary way of thinking and behaving that is shared by the members of
the department. Which of the following terms refer to this?

A. Organizational chart
B. Cultural network
C. Organizational structure
D. Organizational culture

31. He asserts the importance of promoting a positive organizational culture in their unit. Which of the
following behaviors indicate that this is attained by the group?

A. Proactive and caring with one another


B. Competitive and perfectionist
C. Powerful and oppositional
D. Obedient and uncomplaining

32. Stephanie is a new Staff Educator of a private tertiary hospital. She conducts orientation among new
staff nurses in her department. Joseph, one of the new staff nurses, wants to understand the channel of
communication, span of control and lines of communication. Which of the following will provide this
information?

A. Organizational structure
B. Policy
C. Job description
D. Manual of procedures

33. Stephanie is often seen interacting with the medical intern during coffee breaks and after duty hours.
What type of organizational structure is this?

A. Formal
B. Informal
C. Staff
D. Line
9

34. She takes pride in saying that the hospital has a decentralized structure. Which of the following is
NOT compatible with this type of model?

A. Flat organization
B. Participatory approach
C. Shared governance
D. Tall organization

35. Centralized organizations have some advantages. Which of the following statements are TRUE?

1. Highly cost-effective
2. Makes management easier
3. Reflects the interest of the worker
4. Allows quick decisions or actions.

A. 1 & 2
B. 2 & 4
C. 2, 3& 4
D. 1, 2, & 4

36. Stephanie delegates effectively if she has authority to act, which is BEST defined as:

A. having responsibility to direct others


B. being accountable to the organization
C. having legitimate right to act
D. telling others what to do

37. Regardless of the size of a work group, enough staff must be available at all times to accomplish
certain purposes. Which of these purposes is NOT included?

A. Meet the needs of patients


B. Provide a pair of hands to other units as needed
C. Cover all time periods adequately.
D. Allow for growth and development of nursing staff.
10

38. Which of the following guidelines should be least considered in formulating objectives for nursing
care?

A. Written nursing care plan


B. Holistic approach
C. Prescribed standards
D. Staff preferences

39. Stephanie considers shifting to transformational leadership. Which of the following statements best
describes this type of leadership?

A. Uses visioning as the essence of leadership.


B. Serves the followers rather than being served.
C. Maintains full trust and confidence in the subordinates
D. Possesses innate charisma that makes others feel good in his presence.

40. As a manager, she focuses her energy on both the quality of services rendered to the patients as well
as the welfare of the staff of her unit. Which of the following management styles does she adopt?

A. Country club management


B. Organization man management
C. Team management
D. Authority-obedience management
11

Answers and Rationale

Answers and rationale are given below for this exam. Check your correct answers and read the rationales for
additional learning. Tell us your scores on the comments section.

1. Answer: C. Country Club Management

Country club management style puts concern for the staff as number one priority at the expense of
the delivery of services. He/she runs the department just like a country club where everyone is happy including
the manager.

2. Answer: C. Servant leader

Servant leaders are open-minded, listen deeply, try to fully understand others and not being judgmental

3. Answer: A. Possesses inspirational quality that makes followers gets attracted of him and regards him
with reverence

Charismatic leaders make the followers feel at ease in their presence. They feel that they are in good hands
whenever the leader is around.

4. Answer: C. Assessment of personal traits is a reliable tool for predicting a manager’s potential.

It is not conclusive that certain qualities of a person would make him become a good manager. It can only
predict a manager’s potential of becoming a good one.

5. Answer: A. Recognizes staff for going beyond expectations by giving them citations

Path Goal theory according to House and associates rewards good performance so that others would do the
same

6. Answer: C. Great Man

Leaders become leaders because of their birth right. This is also called Genetic theory or the Aristotelian theory
12

7. Answer: C. Laissez faire

Laissez faire leadership is preferred when the followers know what to do and are experts in the field. This
leadership style is relationship-oriented rather than task-centered.

8. Answer: D. Leadership is shared at the point of care.

Shared governance allows the staff nurses to have the authority, responsibility and accountability for their own
practice.

9. Answer: D. Inspires others with vision

Inspires others with a vision is characteristic of a transformational leader. He is focused more on the day-to-day
operations of the department/unit.

10. Answer: A. Have condescending trust and confidence in their subordinates

Benevolent-authoritative managers pretentiously show their trust and confidence to their followers

11. Answer: A. Call for a staff meeting and take this up in the agenda.

This will allow for the participation of every staff in the unit. If they contribute to the solutions of the problem,
they will own the solutions; hence the chance for compliance would be greater.

12. Answer: C. Low morale of staff in his unit

Low morale of staff is an internal factor that affects only the unit. All the rest of the options emanate from the
top executive or from outside the institution.

13. Answer: B. Majority rule

Majority rule involves dividing the house and the highest vote wins. 1/2 + 1 is a majority.

14. Answer: B. system used to deliver care


13

A system used to deliver care. In the 70’s it was termed as methods of patient assignment; in the early 80’s it
was called modalities of patient care then patterns of nursing care in the 90’s until recently authors called it
nursing care systems.

15. Answer: A. Concentrates on tasks and activities

Functional nursing is focused on tasks and activities and not on the holistic care of the patients

16. Answer: A. Psychological and sociological needs are emphasized.

When the functional method is used, the psychological and sociological needs of the patients are neglected; the
patients are regarded as ‘tasks to be done ‘

17. Answer: A. Assessing nursing needs and problems

This option follows the framework of the nursing process at the same time applies the management process of
planning, organizing, directing and controlling

18. Answer: B. Preparing a nursing care plan in collaboration with the patient

The best source of information about the priority needs of the patient is the patient himself. Hence using a
nursing care plan based on his expressed priority needs would ensure meeting his needs effectively.

19. Answer: D. Patients who need the most care

In setting priorities for a group of patients, those who need the most care should be number-one priority to
ensure that their critical needs are met adequately. The needs of other patients who need less care ca be attended
to later or even delegated to assistive personnel according to rules on delegation.

20. Answer: C. Integrate the solutions to his day-to-day activities

Integrate the solutions to his day-to-day activities is expected to happen during the third stage of change when
the change agent incorporate the selected solutions to his system and begins to create a change.

21. Answer: C. Focuses on routine tasks


14

Strategic planning involves options A, B and D except C which is attributed to operational planning

22. Answer: A. The Holy Spirit Medical Center is a trendsetter in tertiary health care in the next five
years

A vision refers to what the institution wants to become within a particular period of time.

23. Answer: B. Goal

24. Answer: C. Broken line

This is a staff relationship hence it is depicted by a broken line in the organizational structure

25. Answer: C. Unity of command

The principle of unity of command means that employees should receive orders coming from only one manager
and not from two managers. This averts the possibility of sowing confusion among the members of the
organization

26. Answer: B. Hierarchy

Hierarchy refers to the pattern of reporting or the formal line of authority in an organizational structure.

27. Answer: B. Unity of direction

Unity of direction means having one goal or one objective for the team to pursue; hence all members of the
organization should put their efforts together towards the attainment of their common goal or objective.

28. Answer: A. “Let’s work together in harmony; we need to be supportive of one another”

The principle of ‘esprit d’ corps’ refers to promoting harmony in the workplace, which is essential in
maintaining a climate conducive to work.

29. Answer: A. Increase the patient satisfaction rate


15

Goal is a desired result towards which efforts are directed.

30. Answer: D. Organizational culture

An organizational culture refers to the way the members of the organization think together and do things around
them together. It’s their way of life in that organization

31. Answer: A. Proactive and caring with one another

32. Answer: A. Organizational structure

Organizational structure provides information on the channel of authority, (i.e., who reports to whom and with
what authority) the number of people who directly reports to the various levels of hierarchy and the lines of
communication whether line or staff.

33. Answer: B. Informal

This is usually not published and oftentimes concealed.

34. Answer: D. Tall organization

Tall organizations are highly centralized organizations where decision making is centered on one authority
level.

35. Answer: A. 1 & 2

Centralized organizations are needs only a few managers hence they are less expensive and easier to manage

36. Answer: C. having legitimate right to act

Authority is a legitimate or official right to give command. This is an officially sanctioned responsibility

37. Answer: B. Provide a pair of hands to other units as needed


16

Providing a pair of hands for other units is not a purpose in doing an effective staffing process. This is a
function of a staffing coordinator at a centralized model.

38. Answer: D. Staff preferences

Staff preferences should be the least priority in formulating objectives of nursing care. Individual preferences
should be subordinate to the interest of the patients.

39. Answer: A. Uses visioning as the essence of leadership.

Transformational leadership relies heavily on visioning as the core of leadership.

40. Answer: C. Team management

Team management has a high concern for services and high concern for staff.
17

Topics or concepts included in this exam are:

 Nursing Leadership and Management


 Leadership
 Delegation
 Chain of Command
 Organizational Structure
 Types of Leaders

1. Katherine is a young Unit Manager of the Pediatric Ward. Most of her staff nurses are senior to her,
very articulate, confident and sometimes aggressive. Katherine feels uncomfortable believing that she is
the scapegoat of everything that goes wrong in her department. Which of the following is the best action
that she must take?

A. Identify the source of the conflict and understand the points of friction
B. Disregard what she feels and continue to work independently
C. Seek help from the Director of Nursing
D. Quit her job and look for another employment.

2. As a young manager, she knows that conflict occurs in any organization. Which of the following
statements regarding conflict is NOT true?

A. Can be destructive if the level is too high


B. Is not beneficial; hence it should be prevented at all times
C. May result in poor performance
D. May create leaders

3. Katherine tells one of the staff, “I don’t have time to discuss the matter with you now. See me in my
office later” when the latter asks if they can talk about an issue. Which of the following conflict resolution
strategies did she use?

A. Smoothing
B. Compromise
C. Avoidance
D. Restriction
18

4. Kathleen knows that one of her staff is experiencing burnout. Which of the following is the best thing
for her to do?

A. Advise her staff to go on vacation.


B. Ignore her observations; it will be resolved even without intervention
C. Remind her to show loyalty to the institution.
D. Let the staff ventilate her feelings and ask how she can be of help.

5. She knows that performance appraisal consists of all the following activities EXCEPT:

A. Setting specific standards and activities for individual performance.


B. Using agency standards as a guide.
C. Determine areas of strength and weaknesses
D. Focusing activity on the correction of identified behavior.

6. Which of the following statements is NOT true about performance appraisal?

A. Informing the staff about the specific impressions of their work help improve their performance.
B. A verbal appraisal is an acceptable substitute for a written report
C. Patients are the best source of information regarding personnel appraisal.
D. The outcome of performance appraisal rests primarily with the staff.

7. There are times when Katherine evaluates her staff as she makes her daily rounds. Which of the
following is NOT a benefit of conducting an informal appraisal?

A. The staff member is observed in natural setting.


B. Incidental confrontation and collaboration is allowed.
C. The evaluation is focused on objective data systematically.
D. The evaluation may provide valid information for compilation of a formal report.

8. She conducts a 6-month performance review session with a staff member. Which of the following
actions is appropriate?

A. She asks another nurse to attest the session as a witness.


B. She informs the staff that she may ask another nurse to read the appraisal before the session is over.
19

C. She tells the staff that the session is manager-centered.


D. The session is private between the two members.

9. Alexandra is tasked to organize the new wing of the hospital. She was given the authority to do as she
deems fit. She is aware that the director of nursing has substantial trust and confidence in her
capabilities, communicates through downward and upward channels and usually uses the ideas and
opinions of her staff. Which of the following is her style of management?

A. Benevolent –authoritative
B. Consultative
C. Exploitive-authoritative
D. Participative

10. She decides to illustrate the organizational structure. Which of the following elements is NOT
included?

A. Level of authority
B. Lines of communication
C. Span of control
D. Unity of direction

11. She plans of assigning competent people to fill the roles designed in the hierarchy. Which process
refers to this?

A. Staffing
B. Scheduling
C. Recruitment
D. Induction

12. She checks the documentary requirements for the applicants for staff nurse position. Which one is
NOT necessary?

A. Certificate of previous employment


B. Record of related learning experience (RLE)
20

C. Membership to accredited professional organization


D. Professional identification card

13. Which phase of the employment process includes getting on the payroll and completing documentary
requirements?

A. Orientation
B. Induction
C. Selection
D. Recruitment

14. She tries to design an organizational structure that allows communication to flow in all directions and
involve workers in decision making. Which form of organizational structure is this?

A. Centralized
B. Decentralized
C. Matrix
D. Informal

15. In a horizontal chart, the lowest level worker is located at the

A. Left most box


B. Middle
C. Right most box
D. Bottom

16. She decides to have a decentralized staffing system. Which of the following is an advantage of this
system of staffing?

A. greater control of activities


B. Conserves time
C. Compatible with computerization
D. Promotes better interpersonal relationship
21

17. Aubrey thinks about primary nursing as a system to deliver care. Which of the following activities is
NOT done by a primary nurse?

A. Collaborates with the physician


B. Provides care to a group of patients together with a group of nurses
C. Provides care for 5-6 patients during their hospital stay.
D. Performs comprehensive initial assessment

18. Which pattern of nursing care involves the care given by a group of paraprofessional workers led by
a professional nurse who take care of patients with the same disease conditions and are located
geographically near each other?

A. Case method
B. Modular nursing
C. Nursing case management
D. Team nursing

19. St. Raphael Medical Center just opened its new Performance Improvement Department. Ms.
Valencia is appointed as the Quality Control Officer. She commits herself to her new role and plans her
strategies to realize the goals and objectives of the department. Which of the following is a primary task
that they should perform to have an effective control system?

A. Make an interpretation about strengths and weaknesses


B. Identify the values of the department
C. Identify structure, process, outcome standards & criteria
D. Measure actual performances

20. Ms. Valencia develops the standards to be followed. Among the following standards, which is
considered as a structure standard?

A. The patients verbalized satisfaction of the nursing care received


B. Rotation of duty will be done every four weeks for all patient care personnel.
C. All patients shall have their weights taken recorded
D. Patients shall answer the evaluation form before discharge
22

21. When she presents the nursing procedures to be followed, she refers to what type of standards?

A. Process
B. Outcome
C. Structure
D. Criteria

22. The following are basic steps in the controlling process of the department. Which of the following is
NOT included?

A. Measure actual performance


B. Set nursing standards and criteria
C. Compare results of performance to standards and objectives
D. Identify possible courses of action

23. Which of the following statements refers to criteria?

A. Agreed on level of nursing care


B. Characteristics used to measure the level of nursing care
C. Step-by-step guidelines
D. Statement which guide the group in decision making and problem solving

24. She wants to ensure that every task is carried out as planned. Which of the following tasks is NOT
included in the controlling process?

A. Instructing the members of the standards committee to prepare policies


B. Reviewing the existing policies of the hospital
C. Evaluating the credentials of all nursing staff
D. Checking if activities conform to schedule

25. Ms. Valencia prepares the process standards. Which of the following is NOT a process standard?

A. Initial assessment shall be done to all patients within twenty four hours upon admission.
B. Informed consent shall be secured prior to any invasive procedure
23

C. Patients’ reports 95% satisfaction rate prior to discharge from the hospital.
D. Patient education about their illness and treatment shall be provided for all patients and their families.

26. Which of the following is evidence that the controlling process is effective?

A. The things that were planned are done


B. Physicians do not complain.
C. Employees are contended
D. There is an increase in customer satisfaction rate.

27. Ms. Valencia is responsible to the number of personnel reporting to her. This principle refers to:

A. Span of control
B. Unity of command
C. Carrot and stick principle
D. Esprit d’ corps

28. She notes that there is an increasing unrest of the staff due to fatiguebrought about by shortage of
staff. Which action is a priority?

A. Evaluate the overall result of the unrest


B. Initiate a group interaction
C. Develop a plan and implement it
D. Identify external and internal forces.

29. Kevin is a member of the Nursing Research Council of the hospital. His first assignment is to
determine the level of patient satisfaction on the care they received from the hospital. He plans to include
all adult patients admitted from April to May, with average length of stay of 3-4 days, first admission,
and with no complications. Which of the following is an extraneous variable of the study?

A. Date of admission
B. Length of stay
C. Age of patients
D. Absence of complications
24

30. He thinks of an appropriate theoretical framework. Whose theory addresses the four modes of
adaptation?

A. Martha Rogers
B. Sr. Callista Roy
C. Florence Nightingale
D. Jean Watson

31. He opts to use a self-report method. Which of the following is NOT TRUE about this method?

A. Most direct means of gathering information


B. Versatile in terms of content coverage
C. Most accurate and valid method of data gathering
D. Yields information that would be difficult to gather by another method

32. Which of the following articles would Kevin least consider for his review of literature?

A. “Story-Telling and Anxiety Reduction Among Pediatric Patients”


B. “Turnaround Time in Emergency Rooms”
C. “Outcome Standards in Tertiary Health Care Institutions”
D. “Environmental Manipulation and Client Outcomes”

33. Which of the following variables will he likely EXCLUDE in his study?

A. Competence of nurses
B. Caring attitude of nurses
C. Salary of nurses
D. Responsiveness of staff

34. He plans to use a Likert Scale to determine

A. degree of agreement and disagreement


B. compliance to expected standards
C. level of satisfaction
D. degree of acceptance
25

35. He checks if his instruments meet the criteria for evaluation. Which of the following criteria refers to
the consistency or the ability to yield the same response upon its repeated administration?

A. Validity
B. Reliability
C. Sensitivity
D. Objectivity

36. Which criteria refer to the ability of the instrument to detect fine differences among the subjects
being studied?

A. Sensitivity
B. Reliability
C. Validity
D. Objectivity

37. Which of the following terms refer to the degree to which an instrument measures what it is supposed
to be measure?

A. Validity
B. Reliability
C. Meaningfulness
D. Sensitivity

38. He plans for his sampling method. Which sampling method gives equal chance to all units in the
population to get picked?

A. Random
B. Accidental
C. Quota
D. Judgment

39. Raphael is interested to learn more about transcultural nursing because he is assigned at the family
suites where most patients come from different cultures and countries. Which of the following designs is
appropriate for this study?
26

A. Grounded theory
B. Ethnography
C. Case study
D. Phenomenology

40. The nursing theorist who developed transcultural nursing theory is

A. Dorothea Orem
B. Madeleine Leininger
C. Betty Newman
D. Sr. Callista Roy
27

Answers and Rationale

Here are the answers and rationales for the exam. This exam is out of this world difficult so don’t feel bad if
you got a low score but be sure to read the rationales below. If you need clarifications, direct them to the
comments below.

1. Answer: A. Identify the source of the conflict and understand the points of friction
This involves a problem solving approach, which addresses the root cause of the problem.

2. Answer: B. Is not beneficial; hence it should be prevented at all times


Conflicts are beneficial because it surfaces out issues in the open and can be solved right away. Likewise,
members of the team become more conscientious with their work when they are aware that other members of
the team are watching them.

3. Answer: C. Avoidance
This strategy shuns discussing the issue head-on and prefers to postpone it to a later time. In effect the problem
remains unsolved and both parties are in a lose-lose situation.

4. Answer: D. Let the staff ventilate her feelings and ask how she can be of help.
Reaching out and helping the staff is the most effective strategy in dealing with burn out. Knowing that
someone is ready to help makes the staff feel important; hence her self-worth is enhanced.

5. Answer: D. Focusing activity on the correction of identified behavior.


Performance appraisal deal with both positive and negative performance; is not meant to be a fault-finding
activity

6. Answer: C. Patients are the best source of information regarding personnel appraisal.
The patient can be a source of information about the performance of the staff but it is never the best source.
Directly observing the staff is the best source of information for personnel appraisal.

7. Answer: C. The evaluation is focused on objective data systematically.


Collecting objective data systematically can not be achieved in an informal appraisal. It is focused on what
actually happens in the natural work setting.
28

8. Answer: D. The session is private between the two members.


The session is private between the manager and the staff and remains to be so when the two parties do not
divulge the information to others.

9. Answer: B. Consultative
A consultative manager is almost like a participative manager. The participative manager has complete trust and
confidence in the subordinate, always uses the opinions and ideas of subordinates and communicates in all
directions.

10. Answer: D. Unity of direction


Unity of direction is a management principle, not an element of an organizational structure.

11. Answer: A. Staffing


Staffing is a management function involving putting the best people to accomplish tasks and activities to attain
the goals of the organization.

12. Answer: B. Record of related learning experience (RLE)


Record of RLE is not required for employment purposes but it is required for the nurse’s licensure examination.

13. Answer: B. Induction


This step in the recruitment process gives time for the staff to submit all the documentary requirements for
employment.

14. Answer: B. Decentralized


Decentralized structures allow the staff to make decisions on matters pertaining to their practice and
communicate in downward, upward, lateral and diagonal flow.

15. Answer: C. Rightmost box


The leftmost box is occupied by the highest authority while the lowest level worker occupies the rightmost box.

16. Answer: D. Promotes better interpersonal relationship


Decentralized structures allow the staff to solve decisions by themselves, involve them in decision making;
hence they are always given opportunities to interact with one another.
29

17. Answer: B. Provides care to a group of patients together with a group of nurses
This function is done in team nursing where the nurse is a member of a team that provides care for a group of
patients.

18. Answer: B. Modular nursing


Modular nursing is a variant of team nursing. The difference lies in the fact that the members in modular
nursing are paraprofessional workers.

19. Answer: B. Identify the values of the department


Identify the values of the department will set the guiding principles within which the department will operate its
activities

20. Answer: B. Rotation of duty will be done every four weeks for all patient care personnel.
Structure standards include management system, facilities, equipment, materials needed to deliver care to
patients. Rotation of duty is a management system.

21. Answer: A. Process


Process standards include care plans, nursing procedure to be done to address the needs of the patients.

22. Answer: D. Identify possible courses of action


This is a step in a quality control process and not a basic step in the control process.

23. Answer: B. Characteristics used to measure the level of nursing care


Criteria are specific characteristics used to measure the standard of care.

24. Answer: A. Instructing the members of the standards committee to prepare policies
Instructing the members involves a directing function.

25. Answer: C. Patients’ reports 95% satisfaction rate prior to discharge from the hospital.
This refers to an outcome standard, which is a result of the care that is rendered to the patient.

26. Answer: A. The things that were planned are done


Controlling is defined as seeing to it that what is planned is done.
30

27. Answer: A. Span of control


Span of control refers to the number of workers who report directly to a manager.

28. Answer: B. Initiate a group interaction


Initiate a group interaction will be an opportunity to discuss the problem in the open.

29. Answer: C. Age of patients


An extraneous variable is not the primary concern of the researcher but has an effect on the results of the study.
Adult patients may be young, middle or late adult.

30. Answer: B. Sr. Callista Roy


Sr. Callista Roy developed the Adaptation Model which involves the physiologic mode, self-concept mode, role
function mode and dependence mode

31. Answer: C. Most accurate and valid method of data gathering


The most serious disadvantage of this method is accuracy and validity of information gathered

32. Answer: B. “Turnaround Time in Emergency Rooms”


The article is for pediatric patients and may not be relevant for adult patients.

33. Answer: C. Salary of nurses


Salary of staff nurses is not an indicator of patient satisfaction, hence need not be included as a variable in the
study.

34. Answer: A. degree of agreement and disagreement


Likert scale is a 5-point summated scale used to determine the degree of agreement or disagreement of the
respondents to a statement in a study.

35. Answer: B. Reliability


Reliability is repeatability of the instrument; it can elicit the same responses even with varied administration of
the instrument

36. Answer: A. Sensitivity


Sensitivity is an attribute of the instrument that allow the respondents to distinguish differences of the options
where to choose from
31

37. Answer: A. Validity


Validity is ensuring that the instrument contains appropriate questions about the research topic

38. Answer: A. Random


Random sampling gives equal chance for all the elements in the population to be picked as part of the sample.

39. Answer: B. Ethnography


Ethnography is focused on patterns of behavior of selected people within a culture

40. Answer: B. Madeleine Leininger


Madeleine Leininger developed the theory on transcultural theory based on her observations on the behavior of
selected people within a culture.
32

NURSING RESEARCH

1. What should be included in “scholarly literature”? Select all that apply.

A. Research reports reported in primary sources only


B. Conceptual and theoretical literature from primary sources only
C. Published and unpublished reports of research
D. Primary and secondary sources

2. Reviews of the literature are conducted for PURPOSES OF RESEARCH as well as for the
CONSUMER OF RESEARCH. How are these reviews similar? Select all that apply.

A. Amount of literature required to be reviewed


B. Degree of critical reading required
C. Importance of conceptual literature
D. Purpose of the review

3. What are characteristics of the literature review required for a quantitative research study? Select all
that apply.

A. The review is exhaustive and must include all studies conducted in the area
B. Doctoral dissertations and masters’ theses are excellent sources of information
C. Computer-accessed materials are acceptable
D. Primary sources are not as important as secondary sources

4. Which of the following is an example of a primary source in a research study?

A. A published commentary on the findings of another study


B. A doctoral dissertation that critiques all research in the area of attention deficit disorder
C. A textbook of medical-surgical nursing
D. A journal article about a study that used large, previously unpublished databases generated by the United
States census

5. What is the best source to use when conducting a level I systematic meta-analysis of the literature?
33

A. An electronic database
B. Doctoral dissertations
C. The Cochrane Statistical Methods
D. An electronic database and Doctoral dissertations

6. What is a characteristic of an audio recording of an unpublished research study reported at a


professional conference?

A. Databased literature
B. Secondary Sources
C. Are more difficult to analyze than written reports.
D. Are not useful because they are not published

7. What is the first step in the qualitative research process?

A. Data analysis
B. Sample
C. Review of literature
D. Study design

8. Which mode of clinical application for qualitative research is considered to be the sharing of
qualitative findings with the patient?

A. Insight or empathy
B. Anticipatory guidance
C. Assessment of status or progress
D. Coaching

9. Which research process steps may be noted in an article’s abstract? Select all that apply.

A. Identifying the phenomenon


B. Research question study purpose
C. Literature review
D. Design
E. Sample
34

F. Legal-ethical issues
G. Data-collection procedure

10. What does a level-of-evidence model use to evaluate the strength of a research study and its findings?
Select all that apply.

A. Creativity
B. Quality
C. Quantity
D. Consistency
E. Efficiency

11. What are the critiquing criteria used to judge the worth of a research study? Select all that apply.

A. Measures
B. Objectives
C. Standards
D. Effectiveness
E. Evaluation guides
F. Questions

12. Which statement best describes qualitative research? Select all that apply.

A. Studies are conducted in natural settings.


B. Data are collected from a large number of subjects.
C. Data collected tend to be numeric.
D. The research design is systematic and subjective.

13. What does a critique of a research study always include? Select all that apply.

A. Determining its strengths and weaknesses


B. Researching similar studies
C. Using critical reading skills
D. Explaining your own personal opinions
35

14. For which of the following research questions would qualitative methods be most appropriate?

A. Which pain medications decrease the need for sleep medication in elderly patients?
B. What is the meaning of health for migrant farm-worker women?
C. Under what conditions does a decubitus ulcer heal most quickly?
D. How does frequency of medication administration impact the degree of painexperienced following knee
replacement surgery?

15. Which of the following phrases would be found in a report of a qualitative study?

A. “The hypothesis of this study is?”


B. “Perceived pain was measured using the Abbott pain scale?”
C. “The control group received no instruction?”
D. “Subjects were asked to relate their perceptions of pain?”

16. Which of the following phrases would be found in a report of a quantitative study?

A. “A convenience sample was chosen?”


B. “The phenomenon studied was?”
C. “Data were analyzed and interpreted?”
D. “Researchers sought to explore the meaning of the hospital experience?”

17. Which of the following hypotheses are indicative of an experimental research design? Select all that
apply.

A. Frequent irrigation of Foley catheters will be positively related to urinary tract infections.
B. The incidence of urinary tract infections will be greater in patients whose Foley catheters are irrigated
frequently than in those whose Foley catheters are irrigated less frequently.
C. Frequent irrigation of Foley catheters is associated with urinary tract infections.
D. The incidence of urinary tract infections will not differ between patients with or without Foley catheters.

18. Which statements are part of the criteria used to judge the soundness of a stated research question?
Select all that apply.
36

A. A relationship between two or more variables


B. An operational definition of each variable
C. The nature of the population being tested
D. The possibility of empirical testing

19. Which criteria are used to determine testability of a hypothesis? Select all that apply.

A. Use of quantifiable words such as greater than or less than


B. A hypothesis stated in such a way that it can be clearly supported or not supported
C. The use of value-laden words in a hypothesis
D. Data-collection efforts that prove the validity of the hypothesis

20. What are the advantages to using directional hypotheses? Select all that apply.

A. The indication of the use of a theory base to derive the hypothesis


B. The provision of a specific theoretical frame of reference
C. Ensurance that findings will be generalizable
D. The indication of a nonbiased selection of subjects
37

Answers and Rationale

1. Answer: C, D

2. Answer: B, C

3. Answer: B, C

4. Answer: D. A journal article about a study that used large, previously unpublished databases
generated by the United States census

This is a report of an original study, so it is the primary source of the study.

5. Answer: C. The Cochrane Statistical Methods

6. Answer: A. Databased literature

Audio and video recordings of research presentations are examples of databased literature.

7. Answer: C. Review of literature

Review of literature is the 1st step in the qualitative research process. Data analysis is the sixth step in the
qualitative research process. Sampling is the third step in the qualitative research process. The study design is
the second step in the qualitative research process.

8. Answer: B. Anticipatory guidance

Anticipatory guidance is the sharing of qualitative findings with the patient. Other options are not considered to
be the sharing of qualitative findings with the patient.

9. Answer: A, B, D

10. Answer: B, C, D

11. Answer: A, C, E, F
38

12. Answer: A, D

13. Answer: A, C

14. Answer: B. What is the meaning of health for migrant farm-worker women?

This question seeks to explore a phenomenon (health) for a specific population.

15. Answer: D. “Subjects were asked to relate their perceptions of pain?”

Data collected were perceptions of pain, not numeric data. Other options are found in a report of a quantitative
study.

16. Answer: A. “A convenience sample was chosen?”

When a sample of convenience is chosen, the study is a quantitative study. Qualitative studies explore
phenomena. Data collected in qualitative studies are “interpreted.” Qualitative studies explore the meaning of
human experience.

17. Answer: B, D

18. Answer: A, C, D

19. Answer: A, B

Quantifiable words increase the testability of a hypothesis (A). The more clearly the hypothesis is stated, the
easier it will be to accept or reject it based on study findings (B). Hypotheses should not have value-laden words
(C). Data-collection processes are not part of the criterion used to evaluate the testability of hypotheses (D).

20. Answer: A, B
39

NURSING RESEARCH

1. Which one of the following statements about hypotheses is most accurate?

A. Hypotheses represent the main idea to be studied and are the foundations of research studies.
B. Hypotheses help frame a test of the validity of a theory.
C. Hypotheses provide the means to test nursing theory.
D. A hypothesis can also be called a problem statement.

2. A nurse wants to study the effectiveness of meditation on people with anxiety disorder. Which variable
would be most relevant to explore in the literature on this topic?

A. Use of meditation during childbirth


B. Meditation techniques found to be effective
C. Pain management for people with anxiety disorders
D. Outcomes of meditation when used by elderly populations

3. What is a characteristic of an independent variable?

A. It is the variable that is predicted to change.


B. It varies with a change in the dependent variable.
C. It is manipulated by the researcher.
D. It can be identified only by changes in the dependent variable.

4. Which statement is most accurate regarding hypotheses?

A. Hypotheses operationally define the dependent variables.


B. Hypotheses are statements about the relationships among variables.
C. Hypotheses describe the effect of the dependent variable on the independent variable.
D. Hypotheses must include a definition of the treatment or intervention used.

5. What is a characteristic of a hypothesis?

A. It flows from interpretation of the data collected.


B. It operationally defines the variable to be studied.
40

C. It eliminates the need to designate a dependent variable.


D. It implies a causative or associative relationship.

6. When should a hypothesis be developed by the researcher during the research process?

A. Before development of the research question


B. After development of the research question
C. After a research design is determined
D. Before any statistical analysis

7. Which research hypothesis is most testable?

A. There is a relationship between meditation and anxiety disorders.


B. Patients with anxiety disorders who learn meditation techniques have less anxiety than those who do not.
C. Teaching one meditation technique to patients with anxiety disorders will be better than teaching multiple
techniques.
D. The ability to meditate causes lower anxiety in patients with anxiety disorder than those who do not
meditate.

8. What is a characteristic of a statistical hypothesis?

A. It is a null hypothesis.
B. It predicts a positive relationship among variables.
C. It is a complex hypothesis.
D. It describes data-analysis methods.

9. When will a null hypothesis be rejected?

A. There is no association among variables.


B. There is evidence of significance.
C. The independent and dependent variables are related.
D. The research hypothesis is rejected.

10. Which level is characteristic of the strength of the evidence provided by the results of a quasi-
experimental study?
41

A. Level I
B. Level II
C. Level III
D. Level IV

11. A researcher wants to discover why patients of certain ethnic backgrounds are reluctant to ask
for pain medication. Because there are little data in the literature on this topic, the researcher designs a
study to explore the relationships between cultural belief systems, the experience of pain, and the
effective use of medication to relieve pain. The researcher plans to use the findings of this study to
formulate hypotheses for a future study. What is a characteristic of this study?

A. It is a quasi-experimental study.
B. It will lead to level II data.
C. It has a directional hypothesis.
D. It is a hypothesis-generating study.

12. The nurse develops the following hypothesis: Elderly women receive less aggressive treatment for
breast cancer than do younger women. Which variable would be considered to be the dependent
variable?

A. Degree of treatment received


B. Age of the patient
C. Type of cancer being treated
D. Use of inpatient treatment

13. The nurse develops the following hypothesis: Elderly women receive less aggressive treatment for
breast cancer than do younger women. Which variable would be considered to be the independent
variable?

A. Degree of treatment received


B. Age of the patient
C. Type of cancer being treated
D. Use of inpatient treatment

14. The following are considered steps in the qualitative research process, except?
42

A. Literature review
B. Hypothesis
C. Sample
D. Data collection

15. Which of the following could be considered the “context” of a study? Select all that apply.

A. Cultural understandings and beliefs of study participants


B. The physical setting of the study
C. The sample selected for the study
D. The number of subjects in the study

16. Which beliefs guide the constructivist paradigm? Select all that apply.

A. There are multiple realities.


B. The truth is objective.
C. Context does not matter as much as truth.
D. The participant (subject) is an active part of the study

17. Which of the following are consistent with the constructivist paradigm? Select all that apply.

A. Subjectivism is valued.
B. Natural laws exist.
C. Time and place are important.
D. Generalizability is valued.

18. Which paradigm provides the basis for qualitative research?

A. Empirical analytical research


B. Constructivism
C. Postpositivism
D. Naturalistic research

19. Which type of research allows researchers to be neutral observers?


43

A. Qualitative research
B. Ethnographic research
C. Quantitative research
D. Case studies

20. Which type of research study can be affected by detracting values of the researcher?

A. Qualitative
B. Naturalistic
C. Ethnographic
D. Quantitative
44

Answers and Rationale

1. Answer: B. Hypotheses help frame a test of the validity of a theory.

Although theories cannot be tested directly, hypotheses provide a bridge between theory and the real world. It is
the research question that represents the main idea to be studied (A). Theories cannot be tested directly (C). The
research question is also called the problem statement (D).

2. Answer: B. Meditation techniques found to be effective

3. Answer: C. It is manipulated by the researcher.

The independent variable is manipulated by the researcher and has a presumed effect on the dependent variable.
It is the dependent variable that is predicted to change (A). The independent variable is presumed to change the
dependent variable (B). The independent variable is manipulated by the researcher and is identified at the
beginning of the study (D).

4. Answer: B. Hypotheses are statements about the relationships among variables.

Hypotheses are statements about the relationships between two or more variables that suggest an answer to the
research question. Hypotheses are not concerned with operationally defining the variables involved in the study
(A). The independent variable is not affected or changed by the dependent variable (C). Hypotheses are not
concerned with operationally defining the variables involved in the study, including treatments or interventions
(D).

5. Answer: D. It implies a causative or associative relationship.

A hypothesis implies a causative or associative relationship. A hypothesis guides the research design and
collection of data (A). Operational definitions are not included in the hypothesis (B). The hypothesis indicates
the dependent variable (C).

6. Answer: B. After development of the research question

The hypothesis is developed after development of the research question.


45

7. Answer: D. The ability to meditate causes lower anxiety in patients with anxiety disorder than those
who do not meditate.

This hypothesis meets the criteria of testability.

8. Answer: A. It is a null hypothesis.

Statistical hypotheses, called null hypotheses, state that there is no relationship between the independent and
dependent variables.

9. Answer: C. The independent and dependent variables are related.

Because the null hypothesis states that there is no relationship between the independent and dependent
variables, it is rejected if they are related.

10. Answer: C. Level III

Evidence provided by quasi-experimental studies is level III. Level I evidence is obtained from a systematic
review of all randomized, controlled trials. Level II evidence is obtained from at least one well-designed
randomized, controlled trial. Level IV evidence is obtained from nonexperimental studies.

11. Answer: D. It is a hypothesis-generating study.

Not enough is known in this area at this time to formulate hypotheses, so the researcher will conduct this
qualitative study and use the findings to generate hypotheses for future studies. This is a qualitative study, not a
quasi-experimental study. Level II evidence is obtained from at least one well-designed randomized, controlled
trial. This study has no hypothesis.

12. Answer: A. Degree of treatment received

The degree of treatment received is considered the dependent variable.

13. Answer: B. Age of the patient

The age of the patient would be the independent variable.


46

14. Answer: B. Hypothesis

A hypothesis is the tool of quantitative studies, and is only found in such studies.

15. Answer: A, B

16. Answer: A, D

17. Answer: A, C

18. Answer: B. Constructivism

The paradigm that provides the basis for qualitative research is constructivism.

19. Answer: C. Quantitative research

In qualitative research, researchers are never considered neutral (A). In ethnography, a type of qualitative
research, researchers are never considered neutral (B). In case studies, a type of qualitative research, researchers
are never considered neutral (D).

20. Answer: D. Quantitative

The values of the researcher must be acknowledged in qualitative research (A). The values of the researcher
must be acknowledged in naturalistic research (B). The values of the researcher must be acknowledged in
qualitative research (C).
47

NURSING RESEARCH

1. What is the purpose of grounded theory?

A. To support theoretical frameworks


B. To generate theory from data
C. To develop explanatory models
D. To find significant differences among groups of people

2. Why is it important to understand the philosophy underlying each type of research?

A. Conclusions reached should be congruent with the research question.


B. The research method that best meets intended purpose of the study should be used.
C. The paradigm of the method should be the same as that of the researcher.
D. The reader should understand the level of abstraction of the study.

3. Which conceptual analysis point of the framework for rigor used for interpretive phenomenology
refers to how the study findings will continue to have meaning for the reader?

A. Resonance
B. Concreteness
C. Actualization
D. Openness

4. Which question will critique the sampling of a research project?

A. Is the strategy used for analysis compatible with the purpose of the study?
B. What is the projected significance of the work to nursing?
C. Are the informants who were chosen appropriate to inform the research?
D. What are the philosophic underpinnings of the research method?

5. Which question will critique the method of a research project?

A. Is the strategy used for analysis compatible with the purpose of the study?
B. What is the projected significance of the work to nursing?
48

C. Are the informants who were chosen appropriate to inform the research?
D. What are the philosophic underpinnings of the research method?

6. Which question will critique the purpose of a research project?

A. Is the strategy used for analysis compatible with the purpose of the study?
B. What is the projected significance of the work to nursing?
C. Are the informants who were chosen appropriate to inform the research?
D. What are the philosophic underpinnings of the research method?

7. Which question will critique the credibility of a research project?

A. Is the strategy used for analysis compatible with the purpose of the study?
B. Does the researcher document the research process?
C. Are the researcher’s conceptualizations true to the data?
D. Has adequate time been allowed to understand fully the phenomenon?

8. Which question will critique the auditability of a research project?

A. Is the strategy used for analysis compatible with the purpose of the study?
B. Does the researcher document the research process?
C. Are the researcher’s conceptualizations true to the data?
D. Has adequate time been allowed to fully understand the phenomenon?

9. Which question will critique the fittingness of a research project?

A. Is the strategy used for analysis compatible with the purpose of the study?
B. Does the researcher document the research process?
C. Are the researcher’s conceptualizations true to the data?
D. Has adequate time been allowed to fully understand the phenomenon?

10. What are the uses of qualitative research methods? Select all that apply.

A. Guiding nursing practice


B. Studying the effects of nursing care on an outcome variable
49

C. Developing survey instruments


D. Developing nursing theory

11. What are scientific criteria appropriate for qualitative research? Select all that apply.

A. Auditability
B. Credibility
C. Fittingness
D. Reliability

12. What are ethical concerns for qualitative researchers? Select all that apply.

A. Because the study emerges over time, the researcher may not anticipate and inform the participants of a
potential threat.
B. To maintain a naturalistic environment for interviews, formal documents such as consent forms are not used.
C. Because there are so few participants in a qualitative study, no participant can opt out of the study.
D. Because the researcher and participant interact over a period of time, relationships developed between them
may change the focus of the interaction

13. Which of the following is most accurate regarding the grounded-theory method?

A. Data are collected using an etic perspective.


B. It is a process of constructing human experience.
C. Secondary sources are sometimes used.
D. It is an inductive approach.

14. What is the term used for the coding and clustering of data to form categories in the grounded-theory
method?

A. Theoretical sampling
B. Constant-comparative method
C. Emic method
D. Metasynthesis

15. What is a characteristic of an intrinsic case study?


50

A. It yields a better understanding of each case.


B. It provides a foundation to challenge a generalization.
C. It does not include quantitative data.
D. It can scrutinize only uncomplicated phenomena.

16. What is a characteristic of metasynthesis?

A. It is useful for triangulating research.


B. It synthesizes critical masses of qualitative findings.
C. It leads to a higher reliability of research findings.
D. It cannot be conducted on historical or case study findings.

17. What is meant by the “fittingness” of a research study?

A. Truth of findings as judged by the participants


B. The appropriateness of the interview questions posed
C. Faithfulness to everyday reality of the participants
D. The adequacy of the coding system used

18. How can qualitative outcome analysis be used? Select all that apply.

A. To determine the reliability of intervention outcomes in a study


B. To confirm the applicability of clinical strategies
C. To develop interventions and then test those selected
D. To build theory

19. When critiquing a qualitative study, which of the following questions are helpful in determining the
study’s auditability? Select all that apply.

A. Has adequate time been allowed to understand the phenomenon fully?


B. Can the reader follow the researcher’s thinking?
C. Are the results meaningful to individuals not involved in the research?
D. Does the researcher document the research process?

20. Which question is helpful in determining the study’s credibility?


51

A. Do the participants recognize the experience as their own?


B. What strategies were used to analyze the data?
C. How were human subjects protected?
D. Are the findings applicable outside the study situation?
52

Answers and Rationale

1. Answer: B. To generate theory from data

The grounded theory method refers to a qualitative approach of building theory about a phenomenon about
which little is known.

2. Answer: B. The research method that best meets intended purpose of the study should be used.

Different research methods accomplish different goals and offer different types and levels of evidence that
inform practice.

3. Answer: C. Actualization

4. Answer: C. Are the informants who were chosen appropriate to inform the research?

5. Answer: A. Is the strategy used for analysis compatible with the purpose of the study?

6. Answer: B. What is the projected significance of the work to nursing?

7. Answer: D. Has adequate time been allowed to understand fully the phenomenon?

8. Answer: B. Does the researcher document the research process?

9. Answer: A. Is the strategy used for analysis compatible with the purpose of the study?

10. Answer: A, C, D

11. Answer: A, B, C

12. Answer: A, D

13. Answer: D. It is an inductive approach.


53

Data are collected using the emic perspective (A). The grounded-theory method is a process of constructing
theory from human experience (B). In grounded-theory only primary sources (the participants) are used (C).

14. Answer: B. Constant-comparative method

15. Answer: A. It yields a better understanding of each case.

An intrinsic case study is undertaken to have a better understanding of the case.

16. Answer: B. It synthesizes critical masses of qualitative findings.

17. Answer: C. Faithfulness to everyday reality of the participants

Credibility is the truth of findings as judged by the participants (A). Auditability assists the reader to judge the
appropriateness of the interview questions posed (B). Auditability assists the reader to judge the adequacy of the
coding system used (D).

18. Answer: B, C, D

19. Answer: B, C

20. Answer: A. Do the participants recognize the experience as their own?


54

NURSING RESEARCH

1. The variable in an experiment that is known from the start


and does not change is called the:

A. dependent variable.

B. extraneous variable.

C. independent variable.

D. confounding variable.

2. Type I errors occur:

A. when the null hypothesis is rejected but it should have been


retained.

B. accepting the null hypothesis when it should have been


rejected.

C. considering the alternate hypothesis as false when it actually


it was true .

D. when the obtained p-value is higher than 0.05.

3. How many degrees of freedom would a table with 3 rows and


2 columns have?

A. 2

B. 1

C. 3

D. 4

4. Determining the Degrees of Freedom for a 2X2 contingency


table for Chi-squire distribution is:

A. 4

B. 2
55

C. 0.05

D. 1

5. The degree of flatness or peakedness of a graph of a


frequency distribution is termed as:

A. standard deviation

B. kurtosis

C. skewness

D. mode

6. In a negatively skewed distribution, the mean generally falls


to:

A. the left of the median and the median usually lies to the left
of the mode.

B. the right of the median and the median usually lies to the
right of the mode.

C. the middle of median and mode.

D. the centre of the distribution.

7. Which statement about normal distribution is FALSE:

A. 50 percent of the observations fall within one standard


deviation sigma of the mean.

B. 68 percent of the observations fall within one standard


deviation sigma of the mean.

C. 95 percent of observation falls within 2 standard deviations.

D. 99.7 percent of observations fall within 3 standard deviations


of the mean.

8. A measure used to standardize the central tendency away


from the mean across different samples is:
56

A. skewness

B. Range

C. Z-score

D. mode

9. Probability values fall on scale between:

A. -1 to +1

B. 0 and 1.

C. -3 to + 3

D. 0.05 to 0.01

10. Standard error is calculated by:

A. dividing standard deviation by the square root of the sample


size.

B. dividing number of nominated outcome by number of


possible outcome.

C. adding all the numbers and then dividing by the numbers of


observations.

D. arranging the numbers in numerical order, then taking the


middle one.

11. When explaining the direction of the linear association


between two numerical paired variables, a positive correlation
is stated when:

A. one variable increases and the other variable decreases or


vice versa.

B. dependent variable increases and independent variable


decreases

C. both variables increase and decrease at the same time.


57

D. correlation coefficient is stated close to 0.

12. A type of graphical presentation data used to explain


correlation between dependent and independent variable is:

A. Histogram

B. Frequency polygon

C. Frequency curve

D. Scatter plot

13. Correlation coefficient ranges from:

A. 0.01 to 0.05

B. 0 to 1

C. -1 to +1

D. -3 to +3

14. A contingency table (2x2) is used to calculate:

A. t-statistic

B. correlation coefficient

C. variance

D. chi-squire statistic

15. The listed observations- 1,2,3,4,100, suggest the distribution:

A. is positively skewed

B. is negatively skewed

C. has zero skewness

D. is left-skewed
58

16. 95% confidence interval refers to:

A. considering 1 out of 20 chances are taken to be wrong.

B. considering 1 out of 100 chances are taken as wrong.

C. considering 95 out of 100 chances are taken as wrong.

D. considering 5 out of 20 chances are taken as wrong.

17. A tentative explanation/statement of a declaration of the


expected outcome of a research study based on observations is
called:

A. Relationships

B. Analysis

C. Variables

D. Hypothesis

18. In a naturalistic observation, the phenomenon in which the


behaviour of the subjects being observed changes because they
are being watched is called:

A. Observer bias

B. Observer effect

C. participant observation

D. Representative sampling

19. The entire group of people or animals in which the


researcher is interested is called:

A. Sample

B. Experiment group

C. Sample

D. Controls
59

20. In the scientific investigation, the first step is:

A. Reporting your results

B. Perceiving a question.

C. Drawing conclusion.

D. Testing hypothesis.

Answer Key

1. C 2. A 3. A 4. D 5. B
6. A 7. A 8. C 9. B 10. A
11. C 12. D 13. C 14. D 15. A
16. A 17. D 18. B 19. C 20. B
60

NURSING RESEARCH

1. Correlation coefficient provides all of the following


information, except :

A. whether or not there is a relationship between the variables.

B. the strength of the relationship between the variables.

C. the cause of the relationship between the variables.

D. the direction of the relationship between the variables.

2. Another name for a bar graph is:

A. polygon

B. histogram

C. normal curve

D. line graph

3. A table that shows how often different scores appear in a set


of scores is called a frequency:

A. polygon

B. histogram

C. normal curve

D. distribution

4. All the following are components required for estimating


power (1-beta) of a study, EXCEPT

A. The population effect size

B. Sample size

C. Level of significance (alpha)

D. Standard deviation
61

5. Using power analysis (Cohen, 1977), the value of estimated


effect size (gamma) for large effects in two group test of mean
difference is

A. 0-2

B. 0.5

C. 0.8

D. 1.0

6. When doing power analysis in ANOVA context, the estimate


of effect size is

A. Eta-squared

B. Correlation coefficient

C. Standard Error

D. Chi-squire value

7. Analysis of Variance (ANOVA) test is used to

A. Test the difference between two independent group means

B. Test the difference between two related group means

C. Test the difference among the means of 3+ independent


groups

D. Test the difference in ranks of scores of 3+ independent


groups

8. Kruskal-Wallis test is used to:

A. Test the difference between two independent group means

B. Test the difference among the means of 3+ related groups


62

C. Test the difference among the means of 3+ independent


groups

D. Test the difference in ranks of scores of 3+ independent


groups

9. Which is the most appropriate inferential analysis to test the


difference among the means of 3+ related groups o sets of
scores?

A. Repeated-measures ANOVA

B. Friedman test

C. Paired t-test

D. Wicoxon signed-rank test

10. Power analysis builds on the concepts of:

A. Validity

B. reliability

C. Internal consistency

D. Effect size

11. Effect size is used to calculate:

A. Reliability

B. Sample size

C. Internal consistency

D. Predictive validity

12. The F-statistic is calculated in:

A. ANOVA

B. T-test
63

C. Correlation analysis

D. Chi-squire test

13. Which of the following is a nonparametric "Analysis of


Variance"?

A. Mann-Whitney U test

B. Wilcoxon Rank test

C. Kruskal-Wallis test

D. Friedman's test

14. Chi-square test is used to test:

A. Difference in proportions

B. Difference in means of two independent variables

C. Relationship between two bivariate variables

D. Difference in means of three or more set of variables

15. Which of the following is NOT a criteria for Fisher's exact


test?

A. When proportions derived from independent groups are


compared

B. When there are cells with a value of 0

C. Sample size is small (less than 30)

D. When testing difference between three or more group mean


Answer Key

1. C 2. B 3.D 4. D 5. C
6. A 7. C 8. D 9. A 10. D
11. B 12. A 13.C 14. A 15. D
64

NURSING RESEARCH

1. The degree of consistency with which an instrument


measures the attribute it is supposed to be measuring is called:

A. Validity

B. Reliability

C. Sensitivity

D. Objectivity

2. The reliability of a measuring tool has following


aspects, except:

A. Stability

B. Internal consistency

C. Efficiency

D. Equivalence

3. The extent to which the same results are obtained on


repeated administrations of the instrument is termed as:

A. Internal consistency

B. Validity

C. Stability

D. Sensitivity

4. The stability index of a measuring tool is derived through


procedures that evaluate:

A. Interrater reliability

B. Internal consistency

C. Crombach's alpha

D. Test-retest reliability
65

5. The Spearman's-Brown formula is used to estimate:

A. Test-retest reliability

B. Internal consistency

C. Equivalence

D. Validity

6. The degree to which an instrument measures what it is


supposed to be measuring is its:

A. Validity

B. Internal consistency

C. Sensitivity

D. Equivalence

7. Which of the following is NOT a type of validity index of a


research instrument?

A. Predictive validity

B. Content validity

C. Concurrent validity

D. Construct validity

E. Homogenous validity

8. The adequacy of an instrument in differentiating between the


performance or behaviour on some future criterion is termed
as:

A. Predictive validity

B. Cocurrent validity

C. Content validity

D. Construct validity
66

9. Multi-trait-multi-matrix (MTMM) is a procedure developed


to establish:

A. Internal consistency

B. Interrater reliability

C. Construct validity

D. Content validity

10. Which of the following is NOT a procedure for establishing


construct validity of an instrument?

A. Known-group technique

B. Factor analysis

C. MTMM

D. Crombach's alpha

11. The most common type of composite scale is:

A. Likert scale

B. Visual analog scale

C. Checklists

D. Thurstone scale

Answer Key

1. B 2.C 3. C 4.D 5.B


6. A 7. E 8. A 9. C 10. D
67

NURSING RESEARCH

1. A sampling process where each element of the population that is sampled is subjected to
an independent Bernoulli trial which determines whether the element becomes part of the sample
during the drawing of a single sample:

A. Simple Random Sampling

B. Stratified Random Sampling

C. Poisson sampling

D. Multistage Sampling

2. Data collection about everyone or everything in group or population and has the advantage of
accuracy and detail:

A. Census

B. Survey

C. Probability sampling

D. Cluster sampling

3. The stages of sampling process comprises: (Multiple Response Question)

A. Defining the population

B. Specifying sampling frame

C. Specifying sampling method

D. Determining the sample size

E. Sampling and data collection


68

4. A sampling method which involves a random start and then proceeds with the selection of every
kth element from then onwards (where k= population size/sample size):

A. Simple random sampling

B. Stratfied random sampling

C. Systematic sampling

D. Snowball sampling

5. Considerations for choosing sample size include:

A. The degree of precision required

B. Method of sampling

C. Way in which results will be analyzed

D. All of the above

6. The standard deviation of a sampling distribution is referred to as the:

A. Test statistic

B. Standard error

C. Confidence interval

D. Test of signaifiance

7. Sampling bias or the error resulting from taking a non-random sample of a population include:

A. Pre-screening

B. Self-Selection Bias

C. Selection from a Specific Area

D. Exclusion bias

E. All of the above


69

Answer Key copyright@ currentnursing.com

1. C 2. A 3. A, B, C, D, E 4. C 5. D

6. B 7. E
70

NURSING RESEARCH

1. "Cardiac patients who receive support from former patients


have less anxiety and higher self-efficacy than other patients".
This statement is an example of:

A. Directional hypothesis=

B. Non-directional hypothesis

C. Statistical hypothesis

D. Null hypothesis

2. What is TRUE about research hypothesis?

A. States there is no relationship between the variables.

B. Statement about the expected relationship of the variables.=

C. States a negative relationship between the variables

D. Research hypothesis should always be directional.

3. Hypothesis testing is sometimes called:

A. Exploratory data analysis

B. Power analysis

C. Deductive Reasoning

D. Confirmatory data analysis =

4. The test's probability of correctly rejecting the null


hypothesis is stated by:

A. Significance level of a test (α)

B. Effect Size

C. Power of a test (1 − β)

D. z = (xbar - µ) / (σ / √n)
71

5. Type I error refers to:

A. Rejecting H0 when it is actually true=

B. Accepting H0 when it is actually false

C. Concluding no difference when one does exist

D. Rejecting research hypothesis when H0 is actually false

6. Which statement is FALSE?

A. H0 and H1 are mutually exclusive

B. Two-tailed tests are more conservative.

C. Probability that the test will lead to a Type I error is Alpha-


level

D. Hypothesis is formulated in experimental research.

E. Probability of falsely rejecting the true null hypothesis (type


II error / alpha error)

7. When considering levels of significance, critical value of z


corresponding to the alpha level of .05 is:

A. .001

B. 1.96

C. 2.58

D. 3.29

Answer Key copyright@ currentnursing.com

1. A 2. B 3. D 4. C 5. A
6. E 7. B
72

NURSING RESEARCH

1. Which of the following classification of research designs is


based on the dimension of control over independent
variables?

A. Structured and flexible

B. Cross-sectional and longitudinal

C. Retrospective and prospective

D. Experimental/quasi experimental and non-experimental

2. A type of longitudinal study in which data are collected


from the same people at two or more points in time:

A. Case control study

B. Panel study

C. Prospective study

D. Retrospective study

3. The process in which each participant has an equal and


known probability of being assigned to either the control or
the experimental group is:

A. Randomization

B. Manipulation

C. Research control

D.

4. The the primary objective of experimental methodology is:

A. Ensure external validity

B. Improve internal validity

C. To eliminate type I error


73

D. To reduce ethical problems

5. A characteristic features of quasi-experimental design are


all, EXCEPT:

A. Experimental and control groups may be nonequivalent

B. Less powerful than true-experimental design

C. Limited confidence in the internal validity of the study

D. Random assignment of units to comparison groups

6. Repeated-measures design is also known as:

A. Crossover design

B. Solomon four-group design

C. Time series design

D. With-in subjects design

7. The process of maneuvering the independent variable so


that its effect on the dependent variable can be observed is:

A. Deductive Reasoning

B. Delimitation

C. Manipulation

D. Meta-analysis
74

A n s w e r K e y copyright@ currentnursing.com

1. D 2. B 3. A 4. B 5. D
6. A 7. C
75

NURSING THEORIES

1. Self-care deficit theory was proposed by:

A. Virginia Henderson

B. Betty Neuman

C. Imogene King

D. Dorothea Orem

2. Which theory defines nursing as the science and practice that


expands adaptive abilities and enhances person and environment
transformation?

A. Goal attainment theory

B. Henderson's definition of nursing

C. Roy's adaptation model

D. Faye Glen Abdelah's theory

3. Typology of twenty one Nursing problems were explained by:

A. Imogene King

B. Virginia Henderson’s

C. Faye G.Abedellah

D. Lydia E. Hall

4. "Nursing is therapeutic interpersonal process". This


definition was stated by:

A. Hildegard Peplau

B. Jean watson

C. Faye Glen Abdelah

D. M. Rogers
76

5. Which of the following statements is related to Florence


Nightingale?

A. Nursing is therapeutic interpersonal process.

B. The role of nursing is to facilitate "the body’s reparative


processes" by manipulating client’s environment.

C. Nursing is the science and practice that expands adaptive


abilities and enhances person and environment transformation

D. Nursing care becomes necessary when client is unable to


fulfill biological, psychological, developmental, or social needs.

6. Which of the following is NOT a concept related to Roy's


Adaptation Model?

A. Focal Stimuli

B. Cognator Subsystem

C. Role function

D. Flexible line of defense

7. According to Roy's adapatation theory, which subsystem


responds through four cognitive responds through four
cognitive-emotive channels (perceptual and information
processing, learning, judgment, and emotion)?

A. Regulator Subsystem

B. Cognator Subsystem

C. Physiologic Mode

D. Self Concept-Group Identity Mode

8. The "humanistic science of nursing" was explained by:

A. Rogers (1970)

B. Ida Orlando (1960)


77

C. Nightingale (1860)

D. Neuman (1972)

9. Imogene King's "goal attainment theory" is a type of:

A. Need theories

B. Interaction theories

C. Outcome theories

D. Humanistic theories

10. Which of the following theory has used "General Systems


Theory" as a framework for its development?

A. Florence Nightingale's Environment Theory

B. Hildegard E. Peplau's Psychodynamic Nursing Theory

C. Martha E.Roger’s: Science of Unitary Human Beings

D. Neuman's model

11. Transcultural Model of Nursing was proposed by:

A. Joyce Travelbee

B. Rosemarie Rizzo Parse

C. Madeleine Leininger

D. Ida Jean Orlando

12. According to Neuman Systems Model, the increase in energy


that occurs in relation to the degree of reaction to the stressor is
termed as:

A. Reconstitution

B. Lines of resistance
78

C. Primary prevention

D. Secondary Prevention

13. Which is NOT a concept explained in Dorothy Johnson's


Behavioral Systems Model?

A. Affiliation

B. Dependency

C. Achievement

D. Energy fields

14. According to Rogers' theory "continuous and mutual


interaction between man and environment' is termed as:

A. Pattern

B. Integrality

C. Resonancy

D. Helicy

15. Watson's carative factors include all the following,


EXCEPT:

A. Forming humanistic-altruistic value system

B. Instilling faith-hope

C. Cultivating sensitivity to self and others

D. Strengthening flexible lines of defense


ANSWER KEY

1. D 2. C 3. C 4. A 5.B
6.D 7.B 8. A 9. B 10. D
11. C 12.A 13. D 14. B 15. D
79

NURSING THEORIES

1. Which is NOT a concept related to Faye Abdellah's theory?

A. Susternal Care Needs

B. The twenty-one Nursing Problems

C. Restorative Care Needs

D. Therapeutic Self-care Demands

2. Statements that explain the relationship between the concepts


in a theory:

A. Propositions

B. Assumptions

C. Predictions

D. Process

3. “Social inclusion, intimacy and the formation and attachment


of a strong social bond” are explained in which subsystem of
Jhonson's model -

A. Dependency subsystem

B. Attachment or affiliative subsystem

C. Achievement subsystem

D. Aggressive subsystem

4. The major concepts of Health Belief Model includes all,


EXCEPT;

A. Perceived Susceptibility

B. Perceived severity

C. Perceived benefits

D. Perceived interaction
80

5. The sequential phases of interpersonal relationship in


Peplau's theory includes all, EXCEPT:

A. Orientation

B. Identification

C. Restoration

D. Exploitation

6. The principles of concervation of energy, structural integrity,


personal integrity and social integrity were explained by:

A. Lydia Hall

B. Myra Estrine Levine

C. Betty Neuman

D. Hildegard Peplau

7. Who explained about "Care, Cure and Core as three


independent but interconnected circles of the nursing model"?

A. Patricia Benner

B. Rosemary Rizzo Parse

C. Lydia Hall

D. Jean Watson
81

8. Meaning, Rhythmicity, Cotranscendence are the three major


concepts of:

A. Transcultural Nursing Theory

B. Unitary Human Being Theory

C. Self-care Deficit Theory

D. Human Becoming Theory

9. " Caring consists of carative factors that result in the


satisfaction of certain human needs". This explanation was
stated by:

A. Sister Calista Roy,

B. Jean Watson

C. Dorothea Orem

D. Florence Nightingale

10. The term which refers the "irreducible, pan dimensional


energy field identified by pattern and integral with the human
field" is:

A. Unitary Human Being

B. Environment

C. Health

D. Nursing

ANSWER KEY

1. D 2. A 3. B 4. D 5. C
6. B 7. C 8. D 9. B 10. B
82

NURSING THEORIES
1. Which of the following nursing theory is based on the general systems framework?

A. Fay Abdellah- Topology of 21 Nursing Problems

B. Virginia Henderson -The Nature of Nursing

C. Hildegard Peplau -Interpersonal Relations Model

D. Imogene King's Theory of Nursing

2. Concept related to Betty Neuman’s System Model of Nursing is:

A. Pattern

B. Rhythmicity

C. Dependency

D. Open system

3. According to Roy's Adaptation Model, the adaptive modes includes all the following, EXCEPT:

A. Physiologic Needs

B. Self Concept

C. Role Function

D. Interdependence

E. Achievement

4. Which theory states " Nursing is a helping profession"?

A. Hildegard Peplau's Interpersonal Theory

B. Abdellah’s 21 Nursing Problems

C. Theory of Goal Attainment

D. Roy's Adaptation Model

5. Which of the following in NOT a concept related to personal system in Imogene King's Theory?

A. Perception

B. Self

C. Body image

D. Organization
83

6. Which nursing theory states that 'nursing is the interpersonal process of action, reaction, interaction and transaction"?

A. Roy's adaptation model

B. Self-care deficit theory

C. Imogene King's theory

D. Roger's unitary human beings

7. All the following are concepts related to Levin's Conservation Principles, EXCEPT:

A. Historicity

B. Specificity

C. Helicy

D. Redundancy

8. Which of the following is an organismic response as per Levin's Four Conservation Principles?

A. Flight or fight

B. Adaptation

C. Communication

D. Transaction

9. When applying Roy's Adaptation Model in caring a patient, the type of stimuli which needs to be assessed as per are all the
following, EXCEPT;

A. Focal Stimulus

B. Contextual Stimulus

C. Perceptual Stimulua

D. Residual Stimulus

10. Who described about 5 levels of nursing experience from novice to expert?

A. Patricia E. Benner

B. Ernestine Wiedenbach

C. Myra Estrine Levine

D. Faye Glenn Abdellah


84

ANSWER KEY
1.D 2. D 3. E 4. B 5. D

6. C 7. C 8.A 9. C 10. A

1. Cognator subsystem is a concept related to:

A. Jhonson's Behaviour System Model

B. Imogene King's Goal Attainment Theory

C. Roy's Adaptation Model

D. Neuman's System's Model

2. Cognator subsystem involves all the following cognitive-emotive channels, EXCEPT:

A. perceptual and information processing

B. self concept

C. learning

D. judgment

E. emotion

3. Each subsystem in Johnson's Behavioural System model is composed of four structural characteristics, except:

A. Drives

B. Set

C. Choices

D. Observable behavior

E. Demands

4. "The practice of activities that individual initiates and perform on their own behalf in maintaining life, health and well being" is:

A. Self care agency

B. Self care

C. Therapeutic self care demand

D. Nursing systems
85

5. Category of self care requisites according to Orem's theory of nursing includes all, except:

A. Universal

B. Developmental

C. Health deviation

D. Fundamental

6.

Nursing is “an external regulatory force which acts to preserve the organization and integration of the patients behaviors at an
optimum level under those conditions in which the behaviors constitutes a threat to the physical or social health, or in which
illness is found”

This definition of nursing was given by

A. Orem

B. Neuman

C. Imogene King

D. Johnson

E. Rogers

7.

"The unique function of the nurse is to assist the individual, sick or well, in the performance of those activities contributing to
health or its recovery (or to peaceful death) that he would perform unaided if he had the necessary strength, will or knowledge. And
to do this in such a way as to help him gain independence as rapidly as possible"

This definition of nursing was given by:

A. Nightingale

B. Neuman

C. Imogene King

D. Henderson

E. Rogers

8. Which level of needs in Maslow’s hierarchy includes love, friendship, intimacy, and family?

A. Self-actualization

B. Esteem

C. Belongingness
86

D. Safety

E. Physiological

9. In a theory, a statement of relationship between concepts is called

A. Conceptual model

B. Hypothesis

C. Proposition

D. Construct

10. Which nursing model was developed by Joyce Travelbee (1926-1973)?

A. Human-to-Human Relationship Model

B. Human becoming theory

C. The theory of health as expanding consciousness

D. From Novice to Expert

ANSWER KEY
1. C 2. B 3. E 4.B 5. D.
6. D 7. D 8. C 9. C 10. A
87

NURSING
1. “Each human being perceives the world as a total person in making transactions with individuals and things in environment”.

This assumption is stated by:

A. Neuman's system model

B. Nightingale's theory

C. Peplau's Interpersonal Relations model

D. Imogene King’s conceptual framework

2. Notes on Nursing: What it is, What it is not was written by:

A. Virginia Henderson

B. Betty Neuman

C. Imogene King

D. Dorothea Orem

E. Florence Nightingale

3. Ernestine Wiedenbach's conceptual model of nursing is called:

A. The Helping Art of Clinical Nursing

B. Modeling and Role Modeling Theory

C. Health As Expanding Consciousness

D. Human-To-Human Relationship Model

4. A system of nursing care in which patients are placed in units on the basis of their needs for care as determined by the degree of
illness rather than on the basis of a medical specialty is:

A. Primary nursing

B. Team nursing

C. Progressive patient care

D. Case method

5. Which of the following terms refers to the branch of philosophy that deals withquestions concerning the nature, scope, and
sources of knowledge?

A. Epistemology
88

B. Epidemiology

C. Metaphysics

D. Ontology

E. Etymology

6. A theory of knowledge emphasizing the role of experience, especially experience based on perceptual observations by
the senses is:

A. Constructivism

B. Empiricism

C. Rationalism

D. Infinitism

7. Metaphysics is a branch of philosophy which deals with :

A. The study of valid argument forms

B. Theory of knowledge

C. Moral philosophy

D. The study of the nature of reality

8. According to Peplau's interpersonal model, during which phase of nursing process, the patient participates in goal setting and
has a feeling of belonging and selectively responds to those who can meet his or her needs?

A. Orientation

B. Identification

C. Exploitation

D. Resolution

9. Deliberative Nursing Process Theory was explained by:

A. Hildegard Peplau

B. Dorothea Orem

C. Ida Jean Orlando

D. Patricia Benner
89

10. Which of the following is NOT a concept related to Nightingale theory?

A. "Poor or difficult environments led to poor health and disease".

B. "Environment could be altered to improve conditions so that the natural laws would allow healing to occur".

C. The goal of nursing is “to put the patient in the best condition for nature to act upon him”.

D. "Human beings are open systems in constant interaction with the environment ".

ANSWER KEY
1. D 2. E 3. A 4. C 5. A
6. B 7. D 8. B 9. C 10. D
90

NURSING THEORIES
1. Which nursing theorist defines environment as "the totality of the internal and external forces which surround a person and with
which they interact at any given time"?

A. Dorothy Johnson

B. Martha Rogers

C. Dorothea Orem

D. Imogene King

E. Betty Neuman

2. The Sunrise Model of nursing was developed by:

A. Joyce Travelbee

B. Rosemarie Rizzo Parse

C. Madeleine Leininger

D. Ida Jean Orlando

3. Ethnonursing research method was developed by:

A. Madeleine Leininger

B. Florence Nightingale

C. Hildegard Peplau

D. Ida Jean Orlando

4. Attachment theory was originally proposed by:

A. Hildegard Peplau

B. John Bowlby

C. Sigmond Frued

D. Kurt Lewin

5. Human Becoming Theory was developed by:

A. Lydia E. Hall

B. Neuman

C. D. Orem
91

D. Rosemary Parse

6. Parse’s Three Principles include all the following, except

A. Helicy

B. Meaning

C. Rhythmicity

D. Cotranscendence

7. Who described 5 levels of nursing experience in her theory on nursing?

A. B. F. Skinner

B. Patricia Benner

C. Callista Roy

D. Leon Festinger

8. A paradigm refers to

A. A model that explains the linkages of science, philosophy, and theory accepted and applied by the discipline

B. Ideas and mental images that help to describe phenomena

C. Statements that describe concepts

D. Aspect of reality that can be consciously sensed or experienced

9. According to Behavour System Model, “predisposition to act with reference to the goal, in certain ways rather than the other
ways” refers to

A. Drive

B. Goal

C. Set

D. Scope of action

10. The study of feedback and derived concepts such as communication and control in living organisms, machines and
organisations is termed as:

A. Cybernetics

B. Ontology

C. Epistemology
92

D. Philosophy

ANSWER KEY
1. E 2. C 3. A 4. B 5. D
6. A 7. B 8. A 9. C 10. A
93

NURSING THEORIES
1. General Systems Theory was first proposed by:

A. Ludwig von Bertalanffy

B. Kurt Lewin

C. Melanie Klein

D. Margaret Mahler

2. Who is a proponent of object relations theory?

A. Abraham Maslaw

B. Jean Piaget

C. Erik Erikson

D. Melanie Klein

3. Which of the following concept is NOT related to Freud's psychoanalytic theory?

A. Phallic stage

B. Pleasure principle

C. Oedipus complex

D. Symbiosis and Separation

4. Kohlberg's theory of moral development includes following stages, except:

A. Obedience and Punishment Orientation

B. Individualism and Exchange

C. Good Interpersonal Relationships

D. Maintaining the Social Order

E. Primary circular reactions

5. The term used to describe the understanding of beliefs, desires, motivations, and emotions as mental states that are ascribed to
one’s self and others: (Tasman 3rd, P 139; 2008)

A. Theory of mind

B. Attribution theory

C. Self-verification theory
94

D. Self-evaluation maintenance theory

6. Which theory explains depression as a process of “turning aggression inward?”

A. Beck's cognitive model

B. Freud's Psychoanalyric model

C. Interpersonal model

D. Engel’s biopsychosocial theory

7. Which theory explains how exposure to trauma that is impossible to avoid may lead to apathy, passivity, and a conviction that
escaping future traumatic events is also impossible?

A. Attribution theory

B. Bowlby's attachment theory

C. Information processing theory

D. Seligman’s theory of learned helplessness

8. Nursing is defined as “action which assist individuals, families and groups to maintain a maximum level of wellness, and the
primary aim is stability of the patient/client system, through nursing interventions to reduce stressors.’’

This definition is given by:

A. Orem

B. Peplau

C. Neuman

D. Rogers

9. According to Piaget, the term used to describe the awareness that objects continue to exist even when they are no longer visible:

A. Oedipal complex

B. Object permanence

C. Theory of mind

D. Inferior complex

10. During cognitive development, object permanence is a characteristic feature of:

A. Sensory-motor stage
95

B. Preoperational stage

C. Concrete operational stage

D. Formal operational stage

ANSWER KEY
1. A 2. D 3. D 4. E 5. A
6. B 7. D 8. C 9. B 10. A
96

NURSING THEORIES
1. A theory that has accrued such persuassive empirical support that is accepted as truth:

A. Rule

B. Model

C. Law

D. Framework

2. A broad theory aimed at describing large segments of the physical, social or behavioural world:

A. Grand theory

B. Middle-range theory

C. Utility theory

D. Philosophy

3. Health is a word symbol that implies forward movement of personality and other ongoing human
processes in the direction of creative, constructive, productive, personal and community living.

This definition of health was given by:

A. Peplau's theory

B. Levin's conservation principles

C. Tidal model of nursing

D. McGill Model

4. "A homeostatic body system is constantly in a dynamic process of input, output, feedback, and
compensation, which leads to a state of balance."

This concept is related to:

A. Peplau

B. Neuman

C. Rogers
97

D. Henderson

5. The process of developing specific predictions from general principles is:

A. Inductive reasoning

B. Deductive reasoning

C. Critical thinking

D. Synectics

6. "What is the nature of reality?"

This question is pertaining to which branch of philosophy?

A. Axiology

B. Epistemology

C. Methodology

D. Ontology

ANSWER KEY
1. C 2. A 3. A 4. B 5. B

6. D
98

PNLE EXAM 1

1. A woman in a child bearing age receives a rubella vaccination. Nurse Joy would give her which of the
following instructions?

A. Refrain from eating eggs or egg products for 24 hours


B. Avoid having sexual intercourse
C. Don’t get pregnant at least 3 months
D. Avoid exposure to sun
2. Jonas who is diagnosed with encephalitis is under the treatment of Mannitol. Which of the following patient
outcomes indicate to Nurse Ronald that the treatment of Mannitol has been effective for a patient that has
increased intracranial pressure?

A. Increased urinary output


B. Decreased RR
C. Slowed papillary response
D. Decreased level of consciousness
3. Mary asked Nurse Maureen about the incubation period of rabies. Which statement by the Nurse Maureen is
appropriate?

A. Incubation period is 6 months


B. Incubation period is 1 week
C. Incubation period is 1 month
D. Incubation period varies depending on the site of the bite
4. Which of the following should Nurse Cherry do first in taking care of a male client with rabies?

A. Encourage the patient to take a bath


B. Cover IV bottle with brown paper bag
C. Place the patient near the comfort room
D. Place the patient near the door
5. Which of the following is the screening test for dengue hemorrhagic fever?

A. Complete blood count


B. ELISA
C. Rumpel-leede test
D. Sedimentation rate
6. Mr. Dela Rosa is suspected to have malaria after a business trip in Palawan. The most important diagnostic
test in malaria is:

A. WBC count
B. Urinalysis
C. ELISA
D. Peripheral blood smear
7. The Nurse supervisor is planning for patient’s assignment for the AM shift. The nurse supervisor avoids
assigning which of the following staff members to a client with herpes zoster?

A. Nurse who never had chicken pox


B. Nurse who never had roseola
C. Nurse who never had german measles
99

D. Nurse who never had mumps


8. Clarissa is 7 weeks pregnant. Further examination revealed that she is susceptible to rubella. When would be
the most appropriate for her to receive rubella immunization?

A. At once
B. During 2nd trimester
C. During 3rd trimester
D. After the delivery of the baby
9. A female child with rubella should be isolated from a:

A. 21 year old male cousin living in the same house


B. 18 year old sister who recently got married
C. 11 year old sister who had rubeola during childhood
D. 4 year old girl who lives next door
10. What is the primary prevention of leprosy?

A. Nutrition
B. Vitamins
C. BCG vaccination
D. DPT vaccination
11. A bacteria which causes diphtheria is also known as?

A. Amoeba
B. Cholera
C. Klebs-loeffler bacillus
D. Spirochete
12. Nurse Ron performed mantoux skin test today (Monday) to a male adult client. Which statement by the
client indicates that he understood the instruction well?

A. I will come back later


B. I will come back next month
C. I will come back on Friday
D. I will come back on Wednesday, same time, to read the result
13. A male client had undergone Mantoux skin test. Nurse Ronald notes an 8mm area of indurations at the site
of the skin test. The nurse interprets the result as:

A. Negative
B. Uncertain and needs to be repeated
C. Positive
D. Inconclusive
14. Tony will start a 6 month therapy with Isoniazid (INH). Nurse Trish plans to teach the client to:

A. Use alcohol moderately


B. Avoid vitamin supplements while o therapy
C. Incomplete intake of dairy products
D. May be discontinued if symptoms subsides
15. Which is the primary characteristic lesion of syphilis?

A. Sore eyes
100

B. Sore throat
C. Chancroid
D. Chancre
16. What is the fast breathing of Jana who is 3 weeks old?

A. 60 breaths per minute


B. 40 breaths per minute
C. 10 breaths per minute
D. 20 breaths per minute
17. Which of the following signs and symptoms indicate some dehydration?

A. Drinks eagerly
B. Restless and irritable
C. Unconscious
D. A and B
18. What is the first line for dysentery?

A. Amoxicillin
B. Tetracycline
C. Cefalexin
D. Cotrimoxazole
19. In home made oresol, what is the ratio of salt and sugar if you want to prepare with 1 liter of water?

A. 1 tbsp. salt and 8 tbsp. sugar


B. 1 tbsp. salt and 8 tsp. sugar
C. 1 tsp. salt and 8 tsp. sugar
D. 8 tsp. salt and 8 tsp. sugar
20. Gentian Violet is used for:

A. Wound
B. Umbilical infections
C. Ear infections
D. Burn
21. Which of the following is a live attenuated bacterial vaccine?

A. BCG
B. OPV
C. Measles
D. None of the above
22. EPI is based on?

A. Basic health services


B. Scope of community affected
C. Epidemiological situation
D. Research studies
23. TT? provides how many percentage of protection against tetanus?

A. 100
B. 99
101

C. 80
D. 90
24. Temperature of refrigerator to maintain potency of measles and OPV vaccine is:

A. -3c to -8c
B. -15c to -25c
C. +15c to +25c
D. +3c to +8c
25. Diptheria is a:

A. Bacterial toxin
B. Killed bacteria
C. Live attenuated
D. Plasma derivatives
26. Budgeting is under in which part of management process?

A. Directing
B. Controlling
C. Organizing
D. Planning
27. Time table showing planned work days and shifts of nursing personnel is:

A. Staffing
B. Schedule
C. Scheduling
D. Planning
28. A force within an individual that influences the strength of behavior?

A. Motivation
B. Envy
C. Reward
D. Self-esteem
29. “To be the leading hospital in the Philippines” is best illustrate in:

A. Mission
B. Philosophy
C. Vision
D. Objective
30. It is the professionally desired norms against which a staff performance will be compared?

A. Job descriptions
B. Survey
C. Flow chart
D. Standards
31. Reprimanding a staff nurse for work that is done incorrectly is an example of what type of reinforcement?

A. Feedback
B. Positive reinforcement
C. Performance appraisal
102

D. Negative reinforcement
32. Questions that are answerable only by choosing an option from a set of given alternatives are known as?

A. Survey
B. Close ended
C. Questionnaire
D. Demographic
33. A researcher that makes a generalization based on observations of an individuals behavior is said to be
which type of reasoning:

A. Inductive
B. Logical
C. Illogical
D. Deductive
34. The balance of a research’s benefit vs. its risks to the subject is:

A. Analysis
B. Risk-benefit ratio
C. Percentile
D. Maximum risk
35. An individual/object that belongs to a general population is a/an:

A. Element
B. Subject
C. Respondent
D. Author
36. An illustration that shows how the members of an organization are connected:

A. Flowchart
B. Bar graph
C. Organizational chart
D. Line graph
37. The first college of nursing that was established in the Philippines is:

A. Fatima University
B. Far Eastern University
C. University of the East
D. University of Sto. Tomas
38. Florence nightingale is born on:

A. France
B. Britain
C. U.S
D. Italy
39. Objective data is also called:

A. Covert
B. Overt
C. Inference
103

D. Evaluation
40. An example of subjective data is:

A. Size of wounds
B. VS
C. Lethargy
D. The statement of patient “My hand is painful”
41. What is the best position in palpating the breast?

A. Trendelenburg
B. Side lying
C. Supine
D. Lithotomy
42. When is the best time in performing breast self examination?

A. 7 days after menstrual period


B. 7 days before menstrual period
C. 5 days after menstrual period
D. 5 days before menstrual period
43. Which of the following should be given the highest priority before performing physical examination to a
patient?

A. Preparation of the room


B. Preparation of the patient
C. Preparation of the nurse
D. Preparation of environment
44. It is a flip over card usually kept in portable file at nursing station.

A. Nursing care plan


B. Medicine and treatment record
C. Kardex
D. TPR sheet
45. Jose has undergone thoracentesis. The nurse in charge is aware that the best position for Jose is:

A. Semi fowlers
B. Low fowlers
C. Side lying, unaffected side
D. Side lying, affected side
46. The degree of patients abdominal distension may be determined by:

A. Auscultation
B. Palpation
C. Inspection
D. Percussion
47. A male client is addicted with hallucinogen. Which physiologic effect should the nurse expect?

A. Bradyprea
B. Bradycardia
C. Constricted pupils
104

D. Dilated pupils
48. Tristan a 4 year old boy has suffered from full thickness burns of the face, chest and neck. What will be the
priority nursing diagnosis?

A. Ineffective airway clearance related to edema


B. Impaired mobility related to pain
C. Impaired urinary elimination related to fluid loss
D. Risk for infection related to epidermal disruption
49. In assessing a client’s incision 1 day after the surgery, Nurse Betty expect to see which of the following as
signs of a local inflammatory response?

A. Greenish discharge
B. Brown exudates at incision edges
C. Pallor around sutures
D. Redness and warmth
50. Nurse Ronald is aware that the amiotic fluid in the third trimester weighs approximately:

A. 2 kilograms
B. 1 kilograms
C. 100 grams
D. 1.5 kilograms
51. After delivery of a baby girl. Nurse Gina examines the umbilical cord and expects to find a cord to:

A. Two arteries and two veins


B. One artery and one vein
C. Two arteries and one vein
D. One artery and two veins
52. Myrna a pregnant client reports that her last menstrual cycle is July 11, her expected date of birth is

A. November 4
B. November 11
C. April 4
D. April 18
53. Which of the following is not a good source of iron?
A. Butter
B. Pechay
C. Grains
D. Beef
54. Maureen is admitted with a diagnosis of ectopic pregnancy. Which of the following would you anticipate?

A. NPO
B. Bed rest
C. Immediate surgery
D. Enema
55. Gina a postpartum client is diagnosed with endometritis. Which position would you expect to place her
based on this diagnosis?

A. Supine
B. Left side lying
105

C. Trendelinburg
D. Semi-fowlers
56. Nurse Hazel knows that Myrna understands her condition well when she remarks that urinary frequency is
caused by:

A. Pressure caused by the ascending uterus


B. Water intake of 3L a day
C. Effect of cold weather
D. Increase intake of fruits and vegetables
57. How many ml of blood is loss during the first 24 hours post delivery of Myrna?

A. 100
B. 500
C. 200
D. 400
58. Which of the following hormones stimulates the secretion of milk?

A. Progesterone
B. Prolactin
C. Oxytocin
D. Estrogen
59. Nurse Carla is aware that Myla’s second stage of labor is beginning when the following assessment is noted:

A. Bay of water is broken


B. Contractions are regular
C. Cervix is completely dilated
D. Presence of bloody show
60. The leaking fluid is tested with nitrazine paper. Nurse Kelly confirms that the client’s membrane have
ruptures when the paper turns into a:

A. Pink
B. Violet
C. Green
D. Blue
61. After amniotomy, the priority nursing action is:

A. Document the color and consistency of amniotic fluid


B. Listen the fetal heart tone
C. Position the mother in her left side
D. Let the mother rest
62. Which is the most frequent reason for postpartum hemorrhage?

A. Perineal lacerations
B. Frequent internal examination (IE)
C. CS
D. Uterine atomy
63. On 2nd postpartum day, which height would you expect to find the fundus in a woman who has had a
caesarian birth?
A. 1 finger above umbilicus
106

B. 2 fingers above umbilicus


C. 2 fingers below umbilicus
D. 1 finger below umbilicus
64. Which of the following criteria allows Nurse Kris to perform home deliveries?

A. Normal findings during assessment


B. Previous CS
C. Diabetes history
D. Hypertensive history
65. Nurse Carla is aware that one of the following vaccines is done by intramuscular (IM) injection?

A. Measles
B. OPV
C. BCG
D. Tetanus toxoid
66. Asin law is on which legal basis:

A. RA 8860
B. RA 2777
C. RI 8172
D. RR 6610
67. Nurse John is aware that the herbal medicine appropriate for urolithiasis is:

A. Akapulco
B. Sambong
C. Tsaang gubat
D. Bayabas
68. Community/Public health bag is defined as:

A. An essential and indispensable equipment of the community health nurse during home visit
B. It contains drugs and equipment used by the community health nurse
C. Is a requirement in the health center and for home visit
D. It is a tool used by the community health nurse in rendering effective procedures during home visit
69. TT4 provides how many percentage of protection against tetanus?

A. 70
B. 80
C. 90
D. 99
70. Third postpartum visit must be done by public health nurse:

A. Within 24 hours after delivery


B. After 2-4 weeks
C. Within 1 week
D. After 2 months
71. Nurse Candy is aware that the family planning method that may give 98% protection to another pregnancy
to women

A. Pills
107

B. Tubal ligation
C. Lactational Amenorrhea method (LAM)
D. IUD
72. Which of the following is not a part of IMCI case management process

A. Counsel the mother


B. Identify the illness
C. Assess the child
D. Treat the child
73. If a young child has pneumonia when should the mother bring him back for follow up?

A. After 2 days
B. In the afternoon
C. After 4 days
D. After 5 days
74. It is the certification recognition program that develop and promotes standard for health facilities:

A. Formula
B. Tutok gamutan
C. Sentrong program movement
D. Sentrong sigla movement
75. Baby Marie was born May 23, 1984. Nurse John will expect finger thumb opposition on:

A. April 1985
B. February 1985
C. March 1985
D. June 1985
76. Baby Reese is a 12 month old child. Nurse Oliver would anticipate how many teeth?

A. 9
B. 7
C. 8
D. 6
77. Which of the following is the primary antidote for Tylenol poisoning?

A. Narcan
B. Digoxin
C. Acetylcysteine
D. Flumazenil
78. A male child has an intelligence quotient of approximately 40. Which kind of environment and
interdisciplinary program most likely to benefit this child would be best described as:

A. Habit training
B. Sheltered workshop
C. Custodial
D. Educational
79. Nurse Judy is aware that following condition would reflect presence of congenital G.I anomaly?

A. Cord prolapse
108

B. Polyhydramios
C. Placenta previa
D. Oligohydramios
80. Nurse Christine provides health teaching for the parents of a child diagnosed with celiac disease. Nurse
Christine teaches the parents to include which of the following food items in the child’s diet:

A. Rye toast
B. Oatmeal
C. White bread
D. Rice
81. Nurse Randy is planning to administer oral medication to a 3 year old child. Nurse Randy is aware that the
best way to proceed is by:

A. “Would you like to drink your medicine?”


B. “If you take your medicine now, I’ll give you lollipop”
C. “See the other boy took his medicine? Now it’s your turn.”
D. “Here’s your medicine. Would you like a mango or orange juice?”
82. At what age a child can brush her teeth without help?

A. 6 years
B. 7 years
C. 5 years
D. 8 years
83. Ribivarin (Virazole) is prescribed for a female hospitalized child with RSV. Nurse Judy prepare this
medication via which route?

A. Intra venous
B. Oral
C. Oxygen tent
D. Subcutaneous
84. The present chairman of the Board of Nursing in the Philippines is:

A. Maria Joanna Cervantes


B. Carmencita Abaquin
C. Leonor Rosero
D. Primitiva Paquic
85. The obligation to maintain efficient ethical standards in the practice of nursing belong to this body:

A. BON
B. ANSAP
C. PNA
D. RN
86. A male nurse was found guilty of negligence. His license was revoked. Re-issuance of revoked certificates
is after how many years?

A. 1 year
B. 2 years
C. 3 years
D. 4 years
109

87. Which of the following information cannot be seen in the PRC identification card?

A. Registration Date
B. License Number
C. Date of Application
D. Signature of PRC chairperson
88. Breastfeeding is being enforced by milk code or:

A. EO 51
B. R.A. 7600
C. R.A. 6700
D. P.D. 996
89. Self governance, ability to choose or carry out decision without undue pressure or coercion from anyone:

A. Veracity
B. Autonomy
C. Fidelity
D. Beneficence
90. A male patient complained because his scheduled surgery was cancelled because of earthquake. The
hospital personnel may be excused because of:

A. Governance
B. Respondent superior
C. Force majeure
D. Res ipsa loquitor
91. Being on time, meeting deadlines and completing all scheduled duties is what virtue?

A. Fidelity
B. Autonomy
C. Veracity
D. Confidentiality
92. This quality is being demonstrated by Nurse Ron who raises the side rails of a confused and disoriented
patient?

A. Responsibility
B. Resourcefulness
C. Autonomy
D. Prudence
93. Which of the following is formal continuing education?

A. Conference
B. Enrollment in graduate school
C. Refresher course
D. Seminar
94. The BSN curriculum prepares the graduates to become?

A. Nurse generalist
B. Nurse specialist
C. Primary health nurse
110

D. Clinical instructor
95. Disposal of medical records in government hospital/institutions must be done in close coordination with
what agency?

A. Department of Health
B. Records Management Archives Office
C. Metro Manila Development Authority
D. Bureau of Internal Revenue
96. Nurse Jolina must see to it that the written consent of mentally ill patients must be taken from:

A. Nurse
B. Priest
C. Family lawyer
D. Parents/legal guardians
97. When Nurse Clarence respects the client’s self-disclosure, this is a gauge for the nurses’

A. Respectfulness
B. Loyalty
C. Trustworthiness
D. Professionalism
98. The Nurse is aware that the following tasks can be safely delegated by the nurse to a non-nurse health
worker except:

A. Taking vital signs


B. Change IV infusions
C. Transferring the client from bed to chair
D. Irrigation of NGT
99. During the evening round Nurse Tina saw Mr. Toralba meditating and afterwards started singing prayerful
hymns. What would be the best response of Nurse Tina?

A. Call the attention of the client and encourage to sleep


B. Report the incidence to head nurse
C. Respect the client’s action
D. Document the situation
100. In caring for a dying client, you should perform which of the following activities

A. Do not resuscitate
B. Assist client to perform ADL
C. Encourage to exercise
D. Assist client towards a peaceful death
101. The Nurse is aware that the ability to enter into the life of another person and perceive his current feelings
and their meaning is known:

A. Belongingness
B. Genuineness
C. Empathy
D. Respect
102. The termination phase of the NPR is best described one of the following:
111

A. Review progress of therapy and attainment of goals


B. Exploring the client’s thoughts, feelings and concerns
C. Identifying and solving patients problem
D. Establishing rapport
103. During the process of cocaine withdrawal, the physician orders which of the following:

A. Haloperidol (Haldol)
B. Imipramine (Tofranil)
C. Benztropine (Cogentin)
D. Diazepam (Valium)
104. The nurse is aware that cocaine is classified as:

A. Hallucinogen
B. Psycho stimulant
C. Anxiolytic
D. Narcotic
105. In community health nursing, it is the most important risk factor in the development of mental illness?

A. Separation of parents
B. Political problems
C. Poverty
D. Sexual abuse
106. All of the following are characteristics of crisis except

A. The client may become resistive and active in stopping the crisis
B. It is self-limiting for 4-6 weeks
C. It is unique in every individual
D. It may also affect the family of the client
107. Freud states that temper tantrums is observed in which of the following:

A. Oral
B. Anal
C. Phallic
D. Latency
108. The nurse is aware that ego development begins during:

A. Toddler period
B. Preschool age
C. School age
D. Infancy
109. Situation: A 19 year old nursing student has lost 36 lbs for 4 weeks. Her parents brought her to the hospital
for medical evaluation. The diagnosis was ANOREXIA NERVOSA. The Primary gain of a client with anorexia
nervosa is:

A. Weight loss
B. Weight gain
C. Reduce anxiety
D. Attractive appearance
110. The nurse is aware that the primary nursing diagnosis for the client is:
112

A. Altered nutrition : less than body requirement


B. Altered nutrition : more than body requirement
C. Impaired tissue integrity
D. Risk for malnutrition
111. After 14 days in the hospital, which finding indicates that her condition in improving?

A. She tells the nurse that she had no idea that she is thin
B. She arrives earlier than scheduled time of group therapy
C. She tells the nurse that she eat 3 times or more in a day
D. She gained 4 lbs in two weeks
112. The nurse is aware that ataractics or psychic energizers are also known as:

A. Anti manic
B. Anti depressants
C. Antipsychotics
D. Anti anxiety
113. Known as mood elevators:

A. Anti depressants
B. Antipsychotics
C. Anti manic
D. Anti anxiety
114. The priority of care for a client with Alzheimer’s disease is

A. Help client develop coping mechanism


B. Encourage to learn new hobbies and interest
C. Provide him stimulating environment
D. Simplify the environment to eliminate the need to make chores
115. Autism is diagnosed at:

A. Infancy
B. 3 years old
C. 5 years old
D. School age
116. The common characteristic of autism child is:

A. Impulsitivity
B. Self destructiveness
C. Hostility
D. Withdrawal
117. The nurse is aware that the most common indication in using ECT is:

A. Schizophrenia
B. Bipolar
C. Anorexia Nervosa
D. Depression
118. A therapy that focuses on here and now principle to promote self-acceptance?

A. Gestalt therapy
113

B. Cognitive therapy
C. Behavior therapy
D. Personality therapy
119. A client has many irrational thoughts. The goal of therapy is to change her:

A. Personality
B. Communication
C. Behavior
D. Cognition
120. The appropriate nutrition for Bipolar I disorder, in manic phase is:

A. Low fat, low sodium


B. Low calorie, high fat
C. Finger foods, high in calorie
D. Small frequent feedings
121. Which of the following activity would be best for a depressed client?

A. Chess
B. Basketball
C. Swimming
D. Finger painting
122. The nurse is aware that clients with severe depression, possess which defense mechanism:

A. Introjection
B. Suppression
C. Repression
D. Projection
123. Nurse John is aware that self mutilation among Bipolar disorder patients is a means of:

A. Overcoming fear of failure


B. Overcoming feeling of insecurity
C. Relieving depression
D. Relieving anxiety
124. Which of the following may cause an increase in the cystitis symptoms?

A. Water
B. Orange juice
C. Coffee
D. Mango juice
125. In caring for clients with renal calculi, which is the priority nursing intervention?

A. Record vital signs


B. Strain urine
C. Limit fluids
D. Administer analgesics as prescribed
126. In patient with renal failure, the diet should be:

A. Low protein, low sodium, low potassium


B. Low protein, high potassium
114

C. High carbohydrate, low protein


D. High calcium, high protein
127. Which of the following cannot be corrected by dialysis?

A. Hypernatremia
B. Hyperkalemia
C. Elevated creatinine
D. Decreased hemoglobin
128. Tony with infection is receiving antibiotic therapy. Later the client complaints of ringing in the ears. This
ototoxicity is damage to:

A. 4th CN
B. 8th CN
C. 7th CN
D. 9th CN
129. Nurse Emma provides teaching to a patient with recurrent urinary tract infection includes the following:

A. Increase intake of tea, coffee and colas


B. Void every 6 hours per day
C. Void immediately after intercourse
D. Take tub bath everyday
130. Which assessment finding indicates circulatory constriction in a male client with a newly applied long leg
cast?

A. Blanching or cyanosis of legs


B. Complaints of pressure or tightness
C. Inability to move toes
D. Numbness of toes
131. During acute gout attack, the nurse administer which of the following drug:

A. Prednisone (Deltasone)
B. Colchicines
C. Aspirin
D. Allopurinol (Zyloprim)
132. Information in the patients chart is inadmissible in court as evidence when:

A. The client objects to its use


B. Handwriting is not legible
C. It has too many unofficial abbreviations
D. The clients parents refuses to use it
133. Nurse Karen is revising a client plan of care. During which step of the nursing process does such revision
take place?

A. Planning
B. Implementation
C. Diagnosing
D. Evaluation
134. When examining a client with abdominal pain, Nurse Hazel should assess:
115

A. Symptomatic quadrant either second or first


B. The symptomatic quadrant last
C. The symptomatic quadrant first
D. Any quadrant
135. How long will nurse John obtain an accurate reading of temperature via oral route?

A. 3 minutes
B. 1 minute
C. 8 minutes
D. 15 minutes
136. The one filing the criminal care against an accused party is said to be the?

A. Guilty
B. Accused
C. Plaintiff
D. Witness
137. A male client has a standing DNR order. He then suddenly stopped breathing and you are at his bedside.
You would:

A. Call the physician


B. Stay with the client and do nothing
C. Call another nurse
D. Call the family
138. The ANA recognized nursing informatics heralding its establishment as a new field in nursing during what
year?

A. 1994
B. 1992
C. 2000
D. 2001
139. When is the first certification of nursing informatics given?

A. 1990-1993
B. 2001-2002
C. 1994-1996
D. 2005-2008
140. The nurse is assessing a female client with possible diagnosis of osteoarthritis. The most significant risk
factor for osteoarthritis is:

A. Obesity
B. Race
C. Job
D. Age
141. A male client complains of vertigo. Nurse Bea anticipates that the client may have a problem with which
portion of the ear?

A. Tymphanic membranes
B. Inner ear
C. Auricle
116

D. External ear
142. When performing Weber’s test, Nurse Rosean expects that this client will hear

A. On unaffected side
B. Longer through bone than air conduction
C. On affected side by bone conduction
D. By neither bone or air conduction
143. Toy with a tentative diagnosis of myasthenia gravis is admitted for diagnostic make up. Myasthenia gravis
can confirmed by:

A. Kernigs sign
B. Brudzinski’s sign
C. A positive sweat chloride test
D. A positive edrophonium (Tensilon) test
144. A male client is hospitalized with Guillain-Barre Syndrome. Which assessment finding is the most
significant?

A. Even, unlabored respirations


B. Soft, non distended abdomen
C. Urine output of 50 ml/hr
D. Warm skin
145. For a female client with suspected intracranial pressure (ICP), a most appropriate respiratory goal is:

A. Maintain partial pressure of arterial oxygen (Pa O2) above 80mmHg


B. Promote elimination of carbon dioxide
C. Lower the PH
D. Prevent respiratory alkalosis
146. Which nursing assessment would identify the earliest sign of ICP?

A. Change in level of consciousness


B. Temperature of over 103°F
C. Widening pulse pressure
D. Unequal pupils
147. The greatest danger of an uncorrected atrial fibrillation for a male patient will be which of the following:

A. Pulmonary embolism
B. Cardiac arrest
C. Thrombus formation
D. Myocardial infarction
148. Linda, A 30 year old post hysterectomy client has visited the health center. She inquired about BSE and
asked the nurse when BSE should be performed. You answered that the BSE is best performed:

A. 7 days after menstruation


B. At the same day each month
C. During menstruation
D. Before menstruation
149. An infant is ordered to recive 500 ml of D5NSS for 24 hours. The Intravenous drip is running at 60
gtts/min. How many drops per minute should the flow rate be?
117

A. 60 gtts/min.
B. 21 gtts/min
C. 30 gtts/min
D. 15 gtts/min
150. Mr. Gutierrez is to receive 1 liter of D5RL to run for 12 hours. The drop factor of the IV infusion set is 10
drops per minute. Approximately how many drops per minutes should the IV be regulated?

A. 13-14 drops
B. 17-18 drops
C. 10-12 drops
D. 15-16 drops
118

Answers
1. C. Don’t get pregnant at least 3 months
2. A. Increased urinary output
3. D. Incubation period varies depending on the site of the bite
4. B. Cover IV bottle with brown paper bag
5. C. Rumpel-leede test
6. D. Peripheral blood smear
7. A. Nurse who never had chicken pox
8. D. After the delivery of the baby
9. B. 18 year old sister who recently got married
10. C. BCG vaccination
11. C. Klebs-loeffler bacillus
12. D. I will come back on Wednesday, same time, to read the result
13. C. Positive
14. B. Avoid vitamin supplements while o therapy
15. D. Chancre
16. A. 60 breaths per minute
17. D. A and B
18. D. Cotrimoxazole
19. C. 1 tsp. salt and 8 tsp. sugar
20. B. Umbilical infections
21. A. BCG
22. C. Epidemiological situation
23. D. 90
24. B. -15c to -25c
25. A. Bacterial toxin
26. D. Planning
27. B. Schedule
28. A. Motivation
29. C. Vision
30. D. Standards
31. D. Negative reinforcement
32. B. Close ended
33. A. Inductive
34. B. Risk-benefit ratio
35. A. Element
36. C. Organizational chart
37. D. University of Sto. Tomas
38. D. Italy
39. B. Overt
40. D. The statement of patient “My hand is painful”
41. C. Supine
42. A. 7 days after menstrual period
43. B. Preparation of the patient
44. C. Kardex
45. C. Side lying, unaffected side
46. D. Percussion
47. D. Dilated pupils
48. A. Ineffective airway clearance related to edema
119

49. D. Redness and warmth


50. B. 1 kilograms
51. C. Two arteries and one vein
52. D. April 18
53. A. Butter
54. C. Immediate surgery
55. D. Semi-fowlers
56. A. Pressure caused by the ascending uterus
57. B. 500
58. D. Estrogen
59. C. Cervix is completely dilated
60. D. Blue
61. B. Listen the fetal heart tone
62. D. Uterine atomy
63. C. 2 fingers below umbilicus
64. A. Normal findings during assessment
65. D. Tetanus toxoid
66. C. RI 8172
67. B. Sambong
68. A. An essential and indispensable equipment of the community health nurse during home visit
69. D. 99
70. B. After 2-4 weeks
71. C. Lactational Amenorrhea method (LAM)
72. B. Identify the illness
73. A. After 2 days
74. D. Sentrong sigla movement
75. B. February 1985
76. D. 6
77. C. Acetylcysteine
78. A. Habit training
79. B. Polyhydramios
80. D. Rice
81. D. “Here’s your medicine. Would you like a mango or orange juice?”
82. A. 6 years
83. C. Oxygen tent
84. B. Carmencita Abaquin
85. A. BON
86. D. 4 years
87. C. Date of Application
88. A. EO 51
89. B. Autonomy
90. C. Force majeure
91. A. Fidelity
92. D. Prudence
93. B. Enrollment in graduate school
94. C. Primary health nurse
95. A. Department of Health
96. D. Parents/legal guardians
97. C. Trustworthiness
98. B. Change IV infusions
120

99. C. Respect the client’s action


100. D. Assist client towards a peaceful death
101. C. Empathy
102. A. Review progress of therapy and attainment of goals
103. D. Diazepam (Valium)
104. B. Psycho stimulant
105. C. Poverty
106. A. The client may become resistive and active in stopping the crisis
107. B. Anal
108. D. Infancy
109. C. Reduce anxiety
110. A. Altered nutrition : less than body requirement
111. D. She gained 4 lbs in two weeks
112. C. Antipsychotics
113. A. Anti depressants
114. D. Simplify the environment to eliminate the need to make chores
115. B. 3 years old
116. D. Withdrawal
117. D. Depression
118. A. Gestalt therapy
119. D. Cognition
120. C. Finger foods, high in calorie
121. D. Finger painting
122. A. Introjection
123. B. Overcoming feeling of insecurity
124. C. Coffee
125. D. Administer analgesics as prescribed
126. A. Low protein, low sodium, low potassium
127. D. Decreased hemoglobin
128. B. 8th CN
129. C. Void immediately after intercourse
130. A. Blanching or cyanosis of legs
131. B. Colchicines
132. A. The client objects to its use
133. D. Evaluation
134. B. The symptomatic quadrant last
135. A. 3 minutes
136. C. Plaintiff
137. B. Stay with the client and do nothing
138. A. 1994
139. B. 2001-2002
140. D. Age
141. B. Inner ear
142. C. On affected side by bone conduction
143. D. A positive edrophonium (Tensilon) test
144. A. Even, unlabored respirations
145. B. Promote elimination of carbon dioxide
146. A. Change in level of consciousness
147. C. Thrombus formation
148. B. At the same day each month
121

149. B. 21 gtts/min
150. A. 13-14 drops
122

PNLE EXAM 2

1. A 10 year old who has sustained a head injury is brought to the emergency department by his
mother. A diagnosis of a mild concussion is made. At the time of discharge, nurse Ron should
instruct the mother to:

A. Withhold food and fluids for 24 hours.


B. Allow him to play outdoors with his friends.
C. Arrange for a follow up visit with the child’s primary care provider in one week.
D. Check for any change in responsiveness every two hours until the follow-up visit.
2. A male client has suffered a motor accident and is now suffering from hypovolemic shock. Nurse
Helen should frequency assess the client’s vital signs during the compensatory stage of shock,
because:

A. Arteriolar constriction occurs


B. The cardiac workload decreases
C. Decreased contractility of the heart occurs
D. The parasympathetic nervous system is triggered
3. A paranoid male client with schizophrenia is losing weight, reluctant to eat, and voicing concerns
about being poisoned. The best intervention by nurse Dina would be to:

A. Allow the client to open canned or pre-packaged food


B. Restrict the client to his room until 2 lbs are gained
C. Have a staff member personally taste all of the client’s food
D. Tell the client the food has been x-rayed by the staff and is safe
4. One day the mother of a young adult confides to nurse Frida that she is very troubled by he
child’s emotional illness. The nurse’s most therapeutic initial response would be:

A. “You may be able to lessen your feelings of guilt by seeking counseling”


B. “It would be helpful if you become involved in volunteer work at this time”
C. “I recognize it’s hard to deal with this, but try to remember that this too shall pass”
D. “Joining a support group of parents who are coping with this problem can be quite helpful.
5. To check for wound hemorrhage after a client has had a surgery for the removal of a tumor in the
neck, nurse grace should:

A. Loosen an edge of the dressing and lift it to see the wound


B. Observe the dressing at the back of the neck for the presence of blood
C. Outline the blood as it appears on the dressing to observe any progression
D. Press gently around the incision to express accumulated blood from the wound
6. A 16-year-old primigravida arrives at the labor and birthing unit in her 38th week of gestation
and states that she is labor. To verify that the client is in true labor nurse Trina should:

A. Obtain sides for a fern test


B. Time any uterine contractions
123

C. Prepare her for a pelvic examination


D. Apply nitrazine paper to moist vaginal tissue
7. As part of the diagnostic workup for pulmonic stenosis, a child has cardiac catheterization. Nurse
Julius is aware that children with pulmonic stenosis have increased pressure:

A. In the pulmonary vein


B. In the pulmonary artery
C. On the left side of the heart
D. On the right side of the heart
8. An obese client asks nurse Julius how to lose weight. Before answering, the nurse should
remember that long-term weight loss occurs best when:

A. Eating patterns are altered


B. Fats are limited in the diet
C. Carbohydrates are regulated
D. Exercise is a major component
9. As a very anxious female client is talking to the nurse May, she starts crying. She appears to be
upset that she cannot control her crying. The most appropriate response by the nurse would be:

A. “Is talking about your problem upsetting you?”


B. “It is Ok to cry; I’ll just stay with you for now”
C. “You look upset; lets talk about why you are crying.”
D. “Sometimes it helps to get it out of your system.”
10. A patient has partial-thickness burns to both legs and portions of his trunk. Which of the
following I.V. fluids is given first?

A. Albumin
B. D5W
C. Lactated Ringer’s solution
D. 0.9% sodium chloride solution with 2 mEq of potassium per 100 ml
11. During the first 48 hours after a severe burn of 40% of the clients body surface, the nurse’s
assessment should include observations for water intoxication. Associated adaptations include:

A. Sooty-colored sputum
B. Frothy pink-tinged sputum
C. Twitching and disorientation
D. Urine output below 30ml per hour
12. After a muscle biopsy, nurse Willy should teach the client to:

A. Change the dressing as needed


B. Resume the usual diet as soon as desired
C. Bathe or shower according to preference
D. Expect a rise in body temperature for 48 hours
124

13. Before a client whose left hand has been amputated can be fitted for a prosthesis, nurse Joy is
aware that:

A. Arm and shoulder muscles must be developed


B. Shrinkage of the residual limb must be completed
C. Dexterity in the other extremity must be achieved
D. Full adjustment to the altered body image must have occurred
14. Nurse Cathy applies a fetal monitor to the abdomen of a client in active labor. When the client
has contractions, the nurse notes a 15 beat per minute deceleration of the fetal heart rate below the
baseline lasting 15 seconds. Nurse Cathy should:

A. Change the maternal position


B. Prepare for an immediate birth
C. Call the physician immediately
D. Obtain the client’s blood pressure
15. A male client receiving prolonged steroid therapy complains of always being thirsty and
urinating frequently. The best initial action by the nurse would be to:

A. Perform a finger stick to test the client’s blood glucose level


B. Have the physician assess the client for an enlarged prostate
C. Obtain a urine specimen from the client for screening purposes
D. Assess the client’s lower extremities for the presence of pitting edema
16. Nurse Bea recognizes that a pacemaker is indicated when a client is experiencing:

A. Angina
B. Chest pain
C. Heart block
D. Tachycardia
17. When administering pancrelipase (Pancreases capsules) to child with cystic fibrosis, nurse Faith
knows they should be given:

A. With meals and snacks


B. Every three hours while awake
C. On awakening, following meals, and at bedtime
D. After each bowel movement and after postural draianage
18. A preterm neonate is receiving oxygen by an overhead hood. During the time the infant is under
the hood, it would be appropriate for nurse Gian to:

A. Hydrate the infant q15 min


B. Put a hat on the infant’s head
C. Keep the oxygen concentration consistent
D. Remove the infant q15 min for stimulation
19. A client’s sputum smears for acid fast bacilli (AFB) are positive, and transmission-based
airborne precautions are ordered. Nurse Kyle should instruct visitors to:
125

A. Limit contact with non-exposed family members


B. Avoid contact with any objects present in the client’s room
C. Wear an Ultra-Filter mask when they are in the client’s room
D. Put on a gown and gloves before going into the client’s room
20. A client with a head injury has a fixed, dilated right pupil; responds only to painful stimuli; and
exhibits decorticate posturing. Nurse Kate should recognize that these are signs of:

A. Meningeal irritation
B. Subdural hemorrhage
C. Medullary compression
D. Cerebral cortex compression
21. After a lateral crushing chest injury, obvious right-sided paradoxic motion of the client’s chest
demonstrates multiple rib fraactures, resulting in a flail chest. The complication the nurse should
carefully observe for would be:

A. Mediastinal shift
B. Tracheal laceration
C. Open pneumothorax
D. Pericardial tamponade
22. When planning care for a client at 30-weeks gestation, admitted to the hospital after vaginal
bleeding secondary to placenta previa, the nurse’s primary objective would be:

A. Provide a calm, quiet environment


B. Prepare the client for an immediate cesarean birth
C. Prevent situations that may stimulate the cervix or uterus
D. Ensure that the client has regular cervical examinations assess for labor
23. When planning discharge teaching for a young female client who has had a pneumothorax, it is
important that the nurse include the signs and symptoms of a pneumothorax and teach the client to
seek medical assistance if she experiences:

A. Substernal chest pain


B. Episodes of palpitation
C. Severe shortness of breath
D. Dizziness when standing up
24. After a laryngectomy, the most important equipment to place at the client’s bedside would be:

A. Suction equipment
B. Humidified oxygen
C. A nonelectric call bell
D. A cold-stream vaporizer
25. Nurse Oliver interviews a young female client with anorexia nervosa to obtain information for
the nursing history. The client’s history is likely to reveal a:

A. Strong desire to improve her body image


126

B. Close, supportive mother-daughter relationship


C. Satisfaction with and desire to maintain her present weight
D. Low level of achievement in school, with little concerns for grades
26. Nurse Bea should plan to assist a client with an obsessive-compulsive disorder to control the
use of ritualistic behavior by:
A. Providing repetitive activities that require little thought
B. Attempting to reduce or limit situations that increase anxiety
C. Getting the client involved with activities that will provide distraction
D. Suggesting that the client perform menial tasks to expiate feelings of guilt
27. A 2 ½ year old child undergoes a ventriculoperitoneal shunt revision. Before discharge, nurse
John, knowing the expected developmental behaviors for this age group, should tell the parents to
call the physician if the child:

A. Tries to copy all the father’s mannerisms


B. Talks incessantly regardless of the presence of others
C. Becomes fussy when frustrated and displays a shortened attention span
D. Frequently starts arguments with playmates by claiming all toys are “mine”
28. A urinary tract infection is a potential danger with an indwelling catheter. Nurse Gina can best
plan to avoid this complication by:

A. Assessing urine specific gravity


B. Maintaining the ordered hydration
C. Collecting a weekly urine specimen
D. Emptying the drainage bag frequently
29. A client has sustained a fractured right femur in a fall on stairs. Nurse Troy with the emergency
response team assess for signs of circulatory impairment by:

A. Turning the client to side lying position


B. Asking the client to cough and deep breathe
C. Taking the client’s pedal pulse in the affected limb
D. Instructing the client to wiggle the toes of the right foot
30. To assess orientation to place in a client suspected of having dementia of the alzheimers type,
nurse Chris should ask:

A. “Where are you?”


B. “Who brought you here?”
C. “Do you know where you are?”
D. “How long have you been there?”
31. Nurse Mary assesses a postpartum client who had an abruption placentae and suspects that
disseminated intravascular coagulation (DIC) is occurring when assessments demonstrate:

A. A boggy uterus
B. Multiple vaginal clots
C. Hypotension and tachycardia
127

D. Bleeding from the venipuncture site


32. When a client on labor experiences the urge to push a 9cm dilation, the breathing pattern that
nurse Rhea should instruct the client to use is the:

A. Expulsion pattern
B. Slow paced pattern
C. Shallow chest pattern
D. blowing pattern
33. Nurse Ronald should explain that the most beneficial between-meal snack for a client who is
recovering from the full-thickness burns would be a:

A. Cheeseburger and a malted


B. Piece of blueberry pie and milk
C. Bacon and tomato sandwich and tea
D. Chicken salad sandwich and soft drink
34. Nurse Wilma recognizes that failure of a newborn to make the appropriate adaptation to
extrauterine life would be indicated by:

A. flexed extremities
B. Cyanotic lips and face
C. A heart rate of 130 beats per minute
D. A respiratory rate of 40 breath per minute
35. The laboratory calls to state that a client’s lithium level is 1.9 mEq/L after 10 days of lithium
therapy. Nurse Reese should:

A. Notify the physician of the findings because the level is dangerously high
B. Monitor the client closely because the level of lithium in the blood is slightly elevated
C. Continue to administer the medication as ordered because the level is within the therapeutic
range
D. Report the findings to the physician so the dosage can be increased because the level is below
therapeutic range
36. A client has a regular 30-day menstrual cycles. When teaching about the rhythm method,
Which the client and her husband have chosen to use for family planning, nurse Dianne should
emphasize that the client’s most fertile days are:

A. Days 9 to 11
B. Days 12 to 14
C. Days 15 to 17
D. Days 18 to 20
37. Before an amniocentesis, nurse Alexandra should:

A. Initiate the intravenous therapy as ordered by the physiscian


B. Inform the client that the procedure could precipitate an infection
C. Assure that informed consent has been obtained from the client
128

D. Perform a vaginal examination on the client to assess cervical dilation


38. While a client is on intravenous magnesium sulfate therapy for preeclampsia, it is essential for
nurse Amy to monitor the client’s deep tendon reflexes to:

A. Determine her level of consciousness


B. Evaluate the mobility of the extremities
C. Determine her response to painful stimuli
D. Prevent development of respiratory distress
39. A preschooler is admitted to the hospital with a diagnosis of acute glomerulonephritis. The
child’s history reveals a 5-pound weight gain in one week and peritoneal edema. For the most
accurate information on the status of the child’s edema, nursing intervention should include:

A. Obtaining the child’s daily weight


B. Doing a visual inspection of the child
C. Measuring the child’s intake and output
D. Monitoring the child’s electrolyte values
40. Nurse Mickey is administering dexamethasome (Decadron) for the early management of a
client’s cerebral edema. This treatment is effective because:

A. Acts as hyperosmotic diuretic


B. Increases tissue resistance to infection
C. Reduces the inflammatory response of tissues
D. Decreases the information of cerebrospinal fluid
41. During newborn nursing assessment, a positive Ortolani’s sign would be indicated by:

A. A unilateral droop of hip


B. A broadening of the perineum
C. An apparent shortening of one leg
D. An audible click on hip manipulation
42. When caring for a dying client who is in the denial stage of grief, the best nursing approach
would be to:

A. Agree and encourage the client’s denial


B. Allow the denial but be available to discuss death
C. Reassure the client that everything will be OK
D. Leave the client alone to confront the feelings of impending loss
43. To decrease the symptoms of gastroesophageal reflux disease (GERD), the physician orders
dietary and medication management. Nurse Helen should teach the client that the meal alteration
that would be most appropriate would be:

A. Ingest foods while they are hot


B. Divide food into four to six meals a day
C. Eat the last of three meals daily by 8pm
D. Suck a peppermint candy after each meal
129

44. After a mastectomy or hysterectomy, clients may feel incomplete as women. The statement that
should alert nurse Gina to this feeling would be:

A. “I can’t wait to see all my friends again”


B. “I feel washed out; there isn’t much left”
C. “I can’t wait to get home to see my grandchild”
D. “My husband plans for me to recuperate at our daughter’s home”
45. A client with obstruction of the common bile duct may show a prolonged bleeding and clotting
time because:

A. Vitamin K is not absorbed


B. The ionized calcium levels falls
C. The extrinsic factor is not absorbed
D. Bilirubin accumulates in the plasma
46. Realizing that the hypokalemia is a side effect of steroid therapy, nurse Monette should monitor
a client taking steroid medication for:

A. Hyperactive reflexes
B. An increased pulse rate
C. Nausea, vomiting, and diarrhea
D. Leg weakness with muscle cramps
47. When assessing a newborn suspected of having Down syndrome, nurse Rey would expect to
observe:

A. long thin fingers


B. Large, protruding ears
C. Hypertonic neck muscles
D. Simian lines on the hands
48. A 10 year old girl is admitted to the pediatric unit for recurrent pain and swelling of her joints,
particularly her knees and ankles. Her diagnosis is juvenile rheumatoid arthritis. Nurse Janah
recognizes that besides joint inflammation, a unique manifestation of the rheumatoid process
involves the:

A. Ears
B. Eyes
C. Liver
D. Brain
49. A disturbed client is scheduled to begin group therapy. The client refuses to attend. Nurse Lolit
should:

A. Accept the client’s decision without discussion


B. Have another client to ask the client to consider
C. Tell the client that attendance at the meeting is required
D. Insist that the client join the group to help the socialization process
130

50. Because a severely depressed client has not responded to any of the antidepressant medications,
the psychiatrist decides to try electroconvulsive therapy (ECT). Before the treatment the nurse
should:

A. Have the client speak with other clients receiving ECT


B. Give the client a detailed explanation of the entire procedure
C. Limit the client’s intake to a light breakfast on the days of the treatment
D. Provide a simple explanation of the procedure and continue to reassure the client
51. Nurse Vicky is aware that teaching about colostomy care is understood when the client states,
“I will contact my physician and report ____”:

A. If I notice a loss of sensation to touch in the stoma tissue”


B. When mucus is passed from the stoma between irrigations”
C. The expulsion of flatus while the irrigating fluid is running out”
D. If I have difficulty in inserting the irrigating tube into the stoma”
52. The client’s history that alerts nurse Henry to assess closely for signs of postpartum infection
would be:

A. Three spontaneous abortions


B. negative maternal blood type
C. Blood loss of 850 ml after a vaginal birth
D. Maternal temperature of 99.9° F 12 hours after delivery
53. A client is experiencing stomatitis as a result of chemotherapy. An appropriate nursing
intervention related to this condition would be to:

A. Provide frequent saline mouthwashes


B. Use karaya powder to decrease irritation
C. Increase fluid intake to compensate for the diarrhea
D. Provide meticulous skin care of the abdomen with Betadine
54. During a group therapy session, one of the clients ask a male client with the diagnosis of
antisocial personality disorder why he is in the hospital. Considering this client’s type of
personality disorder, the nurse might expect him to respond:

A. “I need a lot of help with my troubles”


B. “Society makes people react in old ways”
C. “I decided that it’s time I own up to my problems”
D. “My life needs straightening out and this might help”
55. A child visits the clinic for a 6-week checkup after a tonsillectomy and adenoidectomy. In
addition to assessing hearing, the nurse should include an assessment of the child’s:

A. Taste and smell


B. Taste and speech
C. Swallowing and smell
D. Swallowing and speech
131

56. A client is diagnosed with cancer of the jaw. A course of radiation therapy is to be followed by
surgery. The client is concerned about the side effects related to the radiation treaments. Nurse Ria
should explain that the major side effects that will experienced is:

A. Fatigue
B. Alopecia
C. Vomiting
D. Leucopenia
57. Nurse Katrina prepares an older-adult client for sleep, actions are taken to help reduce the
likelihood of a fall during the night. Targeting the most frequent cause of falls, the nurse should:

A. Offer the client assistance to the bathroom


B. Move the bedside table closer to the client’s bed
C. Encourage the client to take an available sedative
D. Assist the client to telephone the spouse to say “goodnight”
58. When evaluating a growth and development of a 6 month old infant, nurse Patty would expect
the infant to be able to:

A. Sit alone, display pincer grasp, wave bye bye


B. Pull self to a standing position, release a toy by choice, play peek-a-boo
C. Crawl, transfer toy from one hand to the other, display of fear of strangers
D. Turn completely over, sit momentarily without support, reach to be picked up
59. A breastfeeding mother asks the nurse what she can do to ease the discomfort caused by a
cracked nipple. Nurse Tina should instruct the client to:

A. Manually express milk and feed it to the baby in a bottle


B. Stop breastfeeding for two days to allow the nipple to heal
C. Use a breast shield to keep the baby from direct contact with the nipple
D. Feed the baby on the unaffected breast first until the affected breast heals
60. Nurse Sandy observes that there is blood coming from the client’s ear after head injury. Nurse
Sandy should:

A. Turn the client to the unaffected side


B. Cleanse the client’s ear with sterile gauze
C. Test the drainage from the client’s ear with Dextrostix
D. Place sterile cotton loosely in the external ear of the client
61. Nurse Gio plans a long term care for parents of children with sickle-cell anemia, which includes
periodic group conferences. Some of the discussions should be directed towards:

A. Finding special school facilities for the child


B. Making plans for moving to a more therapeutic climate
C. Choosing a means of birth control to avoid future pregnancies
D. Airing their feelings regarding the transmission of the disease to the child
62. The central problem the nurse might face with a disturbed schizophrenic client is the client’s:
132

A. Suspicious feelings
B. Continuous pacing
C. Relationship with the family
D. Concern about working with others
63. When planning care with a client during the postoperative recovery period following an
abdominal hysterectomy and bilateral salpingo-oophorectomy, nurse Frida should include the
explanation that:

A. Surgical menopause will occur


B. Urinary retention is a common problem
C. Weight gain is expected, and dietary plan are needed
D. Depression is normal and should be expected
64. An adolescent client with anorexia nervosa refuses to eat, stating, “I’ll get too fat.” Nurse
Andrea can best respond to this behavior initially by:

A. Not talking about the fact that the client is not eating
B. Stopping all of the client’s priviledges until food is eaten
C. Telling the client that tube feeding will eventually be necessary
D. Pointing out to the client that death can occur with malnutrition.
65. A pain scale is used to assess the degree of pain. The client rates the pain as an 8 on a scale of
10 before medication and a 7 on a scale of 10 after being medicated. Nurse Glenda determines that
the:

A. Client has a low pain tolerance


B. Medication is not adequately effective
C. Medication has sufficiently decreased the pain level
D. Client needs more education about the use of the pain scale
66. To enhance a neonate’s behavioral development, therapeutic nursing measures should include:

A. Keeping the baby awake for longer periods of time before each feeding
B. Assisting the parents to stimulate their baby through touch, sound, and sight.
C. Encouraging parental contact for at least one 15-minute period every four hours.
D. Touching and talking to the baby at least hourly, beginning within two to four hours after birth
67. Before formulating a plan of care for a 6 year old boy with attention deficit hyperactivity
disorder (ADHD), nurse Kyla is aware that the initial aim of therapy is to help the client to:

A. Develop language skills


B. Avoid his own regressive behavior
C. Mainstream into a regular class in school
D. Recognize himself as an independent person of worth
68. Nurse Wally knows that the most important aspect of the preoperative care for a child with
Wilms’ tumor would be:

A. Checking the size of the child’s liver


133

B. Monitoring the child’s blood pressure


C. Maintaining the child in a prone position
D. Collecting the child’s urine for culture and sensitivity
69. At 11:00 pm the count of hydrocodone (Vicodin) is incorrect. After several minutes of
searching the medication cart and medication administration records, no explanation can be found.
The primary nurse should notify the:

A. Nursing unit manager


B. Hospital administrator
C. Quality control manager
D. Physician ordering the medication
70. When caring for the a client with a pneumothorax, who has a chest tube in place, nurse Kate
should plan to:

A. Administer cough suppressants at appropriate intervals as ordered


B. Empty and measure the drainage in the collection chamber each shift
C. Apply clamps below the insertion site when ever getting the client out of bed
D. Encourage coughing, deep breathing, and range of motion to the arm on the affected side
71. According to C.E.Winslow, which of the following is the goal of Public Health?

A. For people to attain their birthrights of health and longevity


B. For promotion of health and prevention of disease
C. For people to have access to basic health services
D. For people to be organized in their health efforts
72. What other statistic may be used to determine attainment of longevity?

A. Age-specific mortality rate


B. Proportionate mortality rate
C. Swaroop’s index
D. Case fatality rate
73. Which of the following is the most prominent feature of public health nursing?

A. It involves providing home care to sick people who are not confined in the hospital
B. Services are provided free of charge to people within the catchment area.
C. The public health nurse functions as part of a team providing a public health nursing services.
D. Public health nursing focuses on preventive, not curative, services.
74. Which of the following is the mission of the Department of Health?

A. Health for all Filipinos


B. Ensure the accessibility and quality of health care
C. Improve the general health status of the population
D. Health in the hands of the Filipino people by the year 2020
75. Nurse Pauline determines whether resources were maximized in implementing Ligtas Tigdas,
she is evaluating:
134

A. Effectiveness
B. Efficiency
C. Adequacy
D. Appropriateness
76. Lissa is a B.S.N. graduate. She want to become a Public Health Nurse. Where will she apply?

A. Department of Health
B. Provincial Health Office
C. Regional Health Office
D. Rural Health Unit
77. As an epidemiologist, Nurse Celeste is responsible for reporting cases of notifiable diseases.
What law mandates reporting of cases of notifiable diseases?

A. Act 3573
B. R.A. 3753
C. R.A. 1054
D. R.A. 1082
78. Nurse Fay is aware that isolation of a child with measles belongs to what level of prevention?

A. Primary
B. Secondary
C. Intermediate
D. Tertiary
79. Nurse Gina is aware that the following is an advantage of a home visit?

A. It allows the nurse to provide nursing care to a greater number of people.


B. It provides an opportunity to do first hand appraisal of the home situation.
C. It allows sharing of experiences among people with similar health problems.
D. It develops the family’s initiative in providing for health needs of its members.
80. The PHN bag is an important tool in providing nursing care during a home visit. The most
important principle of bag technique states that it:

A. Should save time and effort.


B. Should minimize if not totally prevent the spread of infection.
C. Should not overshadow concern for the patient and his family.
D. May be done in a variety of ways depending on the home situation, etc.
81. Nurse Willy reads about Path Goal theory. Which of the following behaviors is manifested by
the leader who uses this theory?

A. Recognizes staff for going beyond expectations by giving them citations


B. Challenges the staff to take individual accountability for their own practice
C. Admonishes staff for being laggards
D. Reminds staff about the sanctions for non performance
135

82. Nurse Cathy learns that some leaders are transactional leaders. Which of the following does
NOT characterize a transactional leader?

A. Focuses on management tasks


B. Is a caretaker
C. Uses trade-offs to meet goals
D. Inspires others with vision
83. Functional nursing has some advantages, which one is an EXCEPTION?

A. Psychological and sociological needs are emphasized.


B. Great control of work activities.
C. Most economical way of delivering nursing services.
D. Workers feel secure in dependent role
84. Which of the following is the best guarantee that the patient’s priority needs are met?

A. Checking with the relative of the patient


B. Preparing a nursing care plan in collaboration with the patient
C. Consulting with the physician
D. Coordinating with other members of the team
85. Nurse Tony stresses the need for all the employees to follow orders and instructions from him
and not from anyone else. Which of the following principles does he refer to?

A. Scalar chain
B. Discipline
C. Unity of command
D. Order
86. Nurse Joey discusses the goal of the department. Which of the following statements is a goal?

A. Increase the patient satisfaction rate


B. Eliminate the incidence of delayed administration of medications
C. Establish rapport with patients
D. Reduce response time to two minutes
87. Nurse Lou considers shifting to transformational leadership. Which of the following statements
best describes this type of leadership?

A. Uses visioning as the essence of leadership


B. Serves the followers rather than being served
C. Maintains full trust and confidence in the subordinates
D. Possesses innate charisma that makes others feel good in his presence.
88. Nurse Mae tells one of the staff, “I don’t have time to discuss the matter with you now. See me
in my office later” when the latter asks if they can talk about an issue. Which of the following
conflict resolution strategies did she use?

A. Smoothing
136

B. Compromise
C. Avoidance
D. Restriction
89. Nurse Bea plans of assigning competent people to fill the roles designed in the hierarchy.
Which process refers to this?

A. Staffing
B. Scheduling
C. Recruitment
D. Induction
90. Nurse Linda tries to design an organizational structure that allows communication to flow in all
directions and involve workers in decision making. Which form of organizational structure is this?

A. Centralized
B. Decentralized
C. Matrix
D. Informal
91. When documenting information in a client’s medical record, the nurse should:

A. erase any errors.


B. use a #2 pencil.
C. leave one line blank before each new entry.
D. end each entry with the nurse’s signature and title.
92. Which of the following factors are major components of a client’s general background drug
history?

A. Allergies and socioeconomic status


B. Urine output and allergies
C. Gastric reflex and age
D. Bowel habits and allergies
93. Which procedure or practice requires surgical asepsis?

A. Hand washing
B. Nasogastric tube irrigation
C. I.V. cannula insertion
D. Colostomy irrigation
94. The nurse is performing wound care using surgical asepsis. Which of the following practices
violates surgical asepsis?

A. Holding sterile objects above the waist


B. Pouring solution onto a sterile field cloth
C. Considering a 1″ (2.5-cm) edge around the sterile field contaminated
D. Opening the outermost flap of a sterile package away from the body
137

95. On admission, a client has the following arterial blood gas (ABG) values: PaO2, 50 mm Hg;
PaCO2, 70 mm Hg; pH, 7.20; HCO3–, 28 mEq/L. Based on these values,
the nurse should formulate which nursing diagnosis for this client?

A. Risk for deficient fluid volume


B. Deficient fluid volume
C. Impaired gas exchange
D. Metabolic acidosis
96. The use of larvivorous fish in malaria control is the basis for which strategy of malaria control?

A. Stream seeding
B. Stream clearing
C. Destruction of breeding places
D. Zooprophylaxis
97. In Integrated Management of Childhood Illness, severe conditions generally require urgent
referral to a hospital. Which of the following severe conditions DOES NOT always require urgent
referral to a hospital?

A. Mastoiditis
B. Severe dehydration
C. Severe pneumonia
D. Severe febrile disease
98. A mother brought her daughter, 4 years old, to the RHU because of cough and colds. Following
the IMCI assessment guide, which of the following is a danger sign that indicates the need for
urgent referral to a hospital?

A. Inability to drink
B. High grade fever
C. Signs of severe dehydration
D. Cough for more than 30 days
99. Food fortification is one of the strategies to prevent micronutrient deficiency conditions. R.A.
8976 mandates fortification of certain food items. Which of the following is among these food
items?

A. Sugar
B. Bread
C. Margarine
D. Filled milk
100. The major sign of iron deficiency anemia is pallor. What part is best examined for pallor?

A. Palms
B. Nailbeds
C. Around the lips
D. Lower conjunctival sac
138

Answers and Rationales


1. C. Check for any change in responsiveness every two hours until the follow-up visit. Signs of
an epidural hematoma in children usually do not appear for 24 hours or more hours; a follow-
up visit usually is arranged for one to two days after the injury.
2. A. Arteriolar constriction occurs.The early compensation of shock is cardiovascular and is seen
in changes in pulse, BP, and pulse pressure; blood is shunted to vital centers, particularly heart
and brain.
3. A. Allow the client to open canned or pre-packaged food. The client’s comfort, safety, and
nutritional status are the priorities; the client may feel comfortable to eat if the food has been
sealed before reaching the mental health facility.
4. D. “Joining a support group of parents who are coping with this problem can be quite
helpful. Taking with others in similar circumstances provides support and allows for sharing of
experiences.
5. B. Observe the dressing at the back of the neck for the presence of blood. Drainage flows by
gravity.
6. C. Prepare her for a pelvic examination. Pelvic examination would reveal dilation and
effacement
7. D. On the right side of the heart. Pulmonic stenosis increases resistance to blood flow, causing
right ventricular hyperthropy; with right ventricular failure there is an increase in pressure on
the right side of the heart.
8. A. Eating patterns are altered. A new dietary regimen, with a balance of foods from the food
pyramid, must be established and continued for weight reduction to occur and be maintained.
9. B. “It is Ok to cry; I’ll just stay with you for now”. This portrays a nonjudgmental attitude that
recognizes the client’s needs.
10. C. Lactated Ringer’s solution. Lactated Ringer’s solution replaces lost sodium and corrects
metabolic acidosis, both of which commonly occur following a burn. Albumin is used as
adjunct therapy, not primary fluid replacement. Dextrose isn’t given to burn patients during the
first 24 hours because it can cause pseudodiabetes. The patient is hyperkalemic from the
potassium shift from the intracellular space to the plasma, so potassium would be detrimental.
11. C. Twitching and disorientation. Excess extracellular fluid moves into cells (water
intoxication); intracellular fluid excess in sensitive brain cells causes altered mental status;
other signs include anorexia nervosa, nausea, vomiting, twitching, sleepiness, and convulsions.
12. B. Resume the usual diet as soon as desired. As long as the client has no nausea or vomiting,
there are no dietary restriction.
13. B. Shrinkage of the residual limb must be completed. Shrinkage of the residual limb, resulting
from reduction of subcutaneous fat and interstitial fluid, must occur for an adequate fit between
the limb and the prosthesis.
14. A. Change the maternal position. Stimulation of the sympathetic nervous system is an initial
response to mild hypoxia that accompanies partial cord compression (umbilical vein) during
contractions; changing the maternal position can alleviate the compression.
15. A. Perform a finger stick to test the client’s blood glucose level. The client has signs of
diabetes, which may result from steroid therapy, testing the blood glucose level is a method of
screening for diabetes, thus gathering more data.
139

16. C. Heart block. This is the primary indication for a pacemaker because there is an interfere
with the electrical conduction system of the heart.
17. A. With meals and snacks. Pancreases capsules must be taken with food and snacks because it
acts on the nutrients and readies them for absorption.
18. B. Put a hat on the infant’s head. Oxygen has cooling effect, and the baby should be kept warm
so that metabolic activity and oxygen demands are not increased.
19. C. Wear an Ultra-Filter mask when they are in the client’s room. Tubercle bacilli are
transmitted through air currents; therefore personal protective equipment such as an Ultra-Filter
mask is necessary.
20. D. Cerebral cortex compression. Cerebral compression affects pyramidal tracts, resulting in
decorticate rigidity and cranial nerve injury, which cause pupil dilation.
21. A.Mediastinal shift. Mediastinal structures move toward the uninjured lung, reducing
oxygenation and venous return.
22. C. Prevent situations that may stimulate the cervix or uterus. Stimulation of the cervix or uterus
may cause bleeding or hemorrhage and should be avoided.
23. C. Severe shortness of breath. This could indicate a recurrence of the pneumothorax as one side
of the lung is inadequate to meet the oxygen demands of the body.
24. A. Suction equipment. Respiratory complications can occur because of edema of the glottis or
injury to the recurrent laryngeal nerve.
25. A. Strong desire to improve her body image. Clients with anorexia nervosa have a disturbed
self image and always see themselves as fat and needing further reducing.
26. B. Attempting to reduce or limit situations that increase anxiety. Persons with high anxiety
levels develop various behaviors to relieve their anxiety; by reducing anxiety, the need for
these obsessive-compulsive action is reduced.
27. C. Becomes fussy when frustrated and displays a shortened attention span. Shortened attention
span and fussy behavior may indicate a change in intracranial pressure and/or shunt
malfunction.
28. B. Maintaining the ordered hydration. Promoting hydration maintains urine production at a
higher rate, which flushes the bladder and prevents urinary stasis and possible infection.
29. C. Taking the client’s pedal pulse in the affected limb. Monitoring a pedal pulse will assess
circulation to the foot.
30. A. “Where are you?”. “Where are you?” is the best question to elicit information about the
client’s orientation to place because it encourages a response that can be assessed.
31. D. Bleeding from the venipuncture site. This indicates a fibrinogenemia; massive clotting in the
area of the separation has resulted in a lowered circulating fibrinogen.
32. D. blowing pattern. Clients should use a blowing pattern to overcome the premature urge to
push.
33. A. Cheeseburger and a malted. Of the selections offered, this is the highest in calories and
protein, which are needed for increased basal metabolic rate and for tissue repair.
34. B. Cyanotic lips and face. Central cyanosis (blue lips and face) indicates lowered oxygenation
of the blood, caused by either decreased lung expansion or right to left shunting of blood.
35. A. Notify the physician of the findings because the level is dangerously high. Levels close to 2
mEq/L are dangerously close to the toxic level; immediate action must be taken.
140

36. C. Days 15 to 17. Ovulation occurs approximately 14 days before the next menses, about the
16th day in 30 day cycle; the 15th to 17th days would be the best time to avoid sexual
intercourse.
37. C. Assure that informed consent has been obtained from the client. An invasive procedure such
as amniocentesis requires informed consent.
38. D. Prevent development of respiratory distress. Respiratory distress or arrest may occur when
the serum level of magnesium sulfate reaches 12 to 15 mg/dl; deep tendon reflexes disappear
when the serum level is 10 to 12 mg/dl; the drug is withheld in the absence of deep tendon
reflexes; the therapeutic serum level is 5 to 8 mg/dl.
39. A. Obtaining the child’s daily weight. Weight monitoring is the most useful means of assessing
fluid balance and changes in the edematous state; 1 liter of fluid weighs about 2.2 pounds.
40. C. Reduces the inflammatory response of tissues. Corticosteroids act to decrease inflammation
which decreases edema.
41. D. An audible click on hip manipulation. With specific manipulation, an audible click may be
heard of felt as he femoral head slips into the acetabulum.
42. B. Allow the denial but be available to discuss death. This does not remove client’s only way
of coping, and it permits future movement through the grieving process when the client is
ready.
43. B. Divide food into four to six meals a day. The volume of food in the stomach should be kept
small to limit pressure on the cardiac sphincter.
44. B. “I feel washed out; there isn’t much left”. The client’s statement infers an emptiness with an
associated loss.
45. A. Vitamin K is not absorbed. Vitamin K, a fat soluble vitamin, is not absorbed from the GI
tract in the absence of bile; bile enters the duodenum via the common bile duct.
46. D. Leg weakness with muscle cramps. Impulse conduction of skeletal muscle is impaired with
decreased potassium levels, muscular weakness and cramps may occur with hypokalemia.
47. D. Simian lines on the hands. This is characteristic finding in newborns with Down syndrome.
48. B. Eyes. Rheumatoid arthritis can cause inflammation of the iris and ciliary body of the eyes
which may lead to blindness.
49. A. Accept the client’s decision without discussion. This is all the nurse can do until trust is
established; facing the client to attend will disrupt the group.
50. D. Provide a simple explanation of the procedure and continue to reassure the client. The nurse
should offer support and use clear, simple terms to allay client’s anxiety.
51. D. If I have difficulty in inserting the irrigating tube into the stoma”. This occurs with stenosis
of the stoma; forcing insertion of the tube could cause injury.
52. C. Blood loss of 850 ml after a vaginal birth. Excessive blood loss predisposes the client to an
increased risk of infection because of decreased maternal resistance; they expected blood loss
is 350 to 500 ml.
53. A. Provide frequent saline mouthwashes. This is soothing to the oral mucosa and helps prevent
infection.
54. B. “Society makes people react in old ways”. The client is incapable of accepting responsibility
for self-created problems and blames society for the behavior.
55. A. Taste and smell. Swelling can obstruct nasal breathing, interfering with the senses of taste
and smell.
141

56. A. Fatigue. Fatigue is a major problem caused by an increase in waste products because of
catabolic processes.
57. A. Offer the client assistance to the bathroom. Statistics indicate that the most frequent cause of
falls by hospitalized clients is getting up or attempting to get up to the bathroom unassisted.
58. D. Turn completely over, sit momentarily without support, reach to be picked up. These
abilities are age-appropriate for the 6 month old child.
59. D. Feed the baby on the unaffected breast first until the affected breast heals. The most
vigorous sucking will occur during the first few minutes of breastfeeding when the infant
would be on the unaffected breast; later suckling is less traumatic.
60. D. Place sterile cotton loosely in the external ear of the client. This would absorb the drainage
without causing further trauma.
61. D. Airing their feelings regarding the transmission of the disease to the child. Discussion with
parents who have children with similar problems helps to reduce some of their discomfort and
guilt.
62. A. Suspicious feelings. The nurse must deal with these feelings and establish basic trust to
promote a therapeutic milieu.
63. A. Surgical menopause will occur. When a bilateral oophorectomy is performed, both ovaries
are excised, eliminating ovarian hormones and initiating response.
64. D. Pointing out to the client that death can occur with malnutrition. The client expects the nurse
to focus on eating, but the emphasis should be placed on feelings rather than actions.
65. B. Medication is not adequately effective. The expected effect should be more than a one point
decrease in the pain level.
66. B. Assisting the parents to stimulate their baby through touch, sound, and sight. Stimuli are
provided via all the senses; since the infant’s behavioral development is enhanced through
parent-infant interactions, these interactions should be encouraged.
67. D. Recognize himself as an independent person of worth. Academic deficits, an inability to
function within constraints required of certain settings, and negative peer attitudes often lead to
low self-esteem.
68. B. Monitoring the child’s blood pressure. Because the tumor is of renal origin, the rennin
angiotensin mechanism can be involved, and blood pressure monitoring is important.
69. A. Nursing unit manager. Controlled substance issues for a particular nursing unit are the
responsibility of that unit’s nurse manager.
70. D. Encourage coughing, deep breathing, and range of motion to the arm on the affected
side. All these interventions promote aeration of the re-expanding lung and maintenance of
function in the arm and shoulder on the affected side.
71. A. For people to attain their birthrights of health and longevity. According to Winslow, all
public health efforts are for people to realize their birthrights of health and longevity.
72. C. Swaroop’s index. Swaroop’s index is the percentage of the deaths aged 50 years or older. Its
inverse represents the percentage of untimely deaths (those who died younger than 50 years).
73. D. Public health nursing focuses on preventive, not curative, services.. The catchment area in
PHN consists of a residential community, many of whom are well individuals who have greater
need for preventive rather than curative services.
74. B. Ensure the accessibility and quality of health care. Ensuring the accessibility and quality of
health care is the primary mission of DOH.
142

75. B. Efficiency. Efficiency is determining whether the goals were attained at the least possible
cost.
76. D. Rural Health Unit. R.A. 7160 devolved basic health services to local government units
(LGU’s ). The public health nurse is an employee of the LGU.
77. A. Act 3573. Act 3573, the Law on Reporting of Communicable Diseases, enacted in 1929,
mandated the reporting of diseases listed in the law to the nearest health station.
78. A. Primary. The purpose of isolating a client with a communicable disease is to protect those
who are not sick (specific disease prevention).
79. B. It provides an opportunity to do first hand appraisal of the home situation. Choice A is not
correct since a home visit requires that the nurse spend so much time with the family. Choice C
is an advantage of a group conference, while choice D is true of a clinic consultation.
80. B. Should minimize if not totally prevent the spread of infection. Bag technique is performed
before and after handling a client in the home to prevent transmission of infection to and from
the client.
81. A. Recognizes staff for going beyond expectations by giving them citations. Path Goal theory
according to House and associates rewards good performance so that others would do the same.
82. D. Inspires others with vision. Inspires others with a vision is characteristic of a
transformational leader. He is focused more on the day-to-day operations of the
department/unit.
83. A. Psychological and sociological needs are emphasized. When the functional method is used,
the psychological and sociological needs of the patients are neglected; the patients are regarded
as ‘tasks to be done”
84. B. Preparing a nursing care plan in collaboration with the patient. The best source of
information about the priority needs of the patient is the patient himself. Hence using a nursing
care plan based on his expressed priority needs would ensure meeting his needs effectively.
85. C. Unity of command. The principle of unity of command means that employees should
receive orders coming from only one manager and not from two managers. This averts the
possibility of sowing confusion among the members of the organization.
86. A. Increase the patient satisfaction rate. Goal is a desired result towards which efforts are
directed. Options AB, C and D are all objectives which are aimed at specific end.
87. A. Uses visioning as the essence of leadership. Transformational leadership relies heavily on
visioning as the core of leadership.
88. C. Avoidance. This strategy shuns discussing the issue head-on and prefers to postpone it to a
later time. In effect the problem remains unsolved and both parties are in a lose-lose situation.
89. A. Staffing. Staffing is a management function involving putting the best people to accomplish
tasks and activities to attain the goals of the organization.
90. B. Decentralized. Decentralized structures allow the staff to make decisions on matters
pertaining to their practice and communicate in downward, upward, lateral and diagonal flow.
91. D. end each entry with the nurse’s signature and title. The end of each entry should include the
nurse’s signature and title; the signature holds the nurse accountable for the recorded
information. Erasing errors in documentation on a legal document such as a client’s chart isn’t
permitted by law. Because a client’s medical record is considered a legal document, the nurse
should make all entries in ink. The nurse is accountable for the information recorded and
143

therefore shouldn’t leave any blank lines in which another health care worker could make
additions.
92. A. Allergies and socioeconomic status. General background data consist of such components as
allergies, medical history, habits, socioeconomic status, lifestyle, beliefs, and sensory deficits.
Urine output, gastric reflex, and bowel habits are significant only if a disease affecting these
functions is present.
93. C. I.V. cannula insertion. Caregivers must use surgical asepsis when performing wound care or
any procedure in which a sterile body cavity is entered or skin integrity is broken. To achieve
surgical asepsis, objects must be rendered or kept free of all pathogens. Inserting an I.V.
cannula requires surgical asepsis because it disrupts skin integrity and involves entry into a
sterile cavity (a vein). The other options are used to ensure medical asepsis or clean technique
to prevent the spread of infection. The GI tract isn’t sterile; therefore, irrigating a nasogastric
tube or a colostomy requires only clean technique.
94. B. Pouring solution onto a sterile field cloth. Pouring solution onto a sterile field cloth violates
surgical asepsis because moisture penetrating the cloth can carry microorganisms to the sterile
field via capillary action. The other options are practices that help ensure surgical asepsis.
95. C. Impaired gas exchange. The client has a below-normal value for the partial pressure of
arterial oxygen (PaO2) and an above-normal value for the partial pressure of arterial carbon
dioxide (PaCO2), supporting the nursing diagnosis of Impaired gas exchange. ABG values
can’t indicate a diagnosis of Fluid volume deficit (or excess) or Risk for deficient fluid volume.
Metabolic acidosis is a medical, not nursing, diagnosis; in any event, these ABG values
indicate respiratory, not metabolic, acidosis.
96. A. Stream seeding. Stream seeding is done by putting tilapia fry in streams or other bodies of
water identified as breeding places of the Anopheles mosquito.
97. B. Severe dehydration. The order of priority in the management of severe dehydration is as
follows: intravenous fluid therapy, referral to a facility where IV fluids can be initiated within
30 minutes, Oresol/nasogastric tube, Oresol/orem. When the foregoing measures are not
possible or effective, tehn urgent referral to the hospital is done.
98. A. Inability to drink. A sick child aged 2 months to 5 years must be referred urgently to a
hospital if he/she has one or more of the following signs: not able to feed or drink, vomits
everything, convulsions, abnormally sleepy or difficult to awaken.
99. A. Sugar. R.A. 8976 mandates fortification of rice, wheat flour, sugar and cooking oil with
Vitamin A, iron and/or iodine.
100. A. Palms. The anatomic characteristics of the palms allow a reliable and convenient basis for
examination for pallor.

Das könnte Ihnen auch gefallen